¿Cómo estudiar matemáticas para entenderlas realmente y tener un estilo de vida saludable con tiempo libre? [cerrado]

Aquí está mi problema que enfrenté; Trabajé muy duro estudiando Matemáticas, así que por eso empecé a darme cuenta de que entiendo mejor las cosas. Sin embargo, eso tiene un gran costo: en los últimos años, prácticamente no hice ejercicio físico, he ganado 30  kg, pasé innumerables horas estudiando por la noche, tuve privación de sueño constante, perdí mi vida social y desarrollé problemas de salud. Mis calificaciones son bastante buenas, pero siento que estoy desperdiciando mi vida.

Me encantan las matemáticas cuando se hacen a mi manera, pero eso casi nunca. Me gustaría mucho que mi carrera se centrara en las matemáticas (topología, álgebra o algo similar). Quiero entender realmente las cosas y quiero que las pruebas se hagan de una manera (razonablemente) rigurosa. Antes me han acusado de formalista pero no me considero tal en absoluto. Sin embargo, reconozco que soy un perfeccionista. A modo de comparación, las respuestas de Theo, Arturo, Jim Belk, Mariano, etc. son lo suficientemente rigurosas para mí. De acuerdo con mi experiencia, 80 El % o más de las matemáticas en nuestra escuela se hace de una manera incompleta, "Hmm, probablemente sea cierto" (al igual que leer recetas de cocina), lo que me molesta muchísimo. La mayoría de los compañeros de clase se adaptan pero, por alguna razón, yo no puedo. No entiendo las cosas a menos que las entienda (casi) completamente. Aprendieron "cómo se deben hacer las cosas", pero con menos frecuencia se preguntan POR QUÉ es esto correcto. Tengo dos amigos físicos que tienen exactamente el mismo problema. Uno está en el nivel de doctorado, constantemente frustrado, mientras que el otro abandonó la física por completo después de obtener un diploma. Aparte de uno 8 , tenia un record perfecto, todos son 10 s. Mencionó que no siente que entienda la física lo suficientemente bien. Desde mi experiencia, TODOS sus compañeros de clase entienden menos que él, simplemente siguen la corriente y aceptan ciertas afirmaciones como verdaderas. ¿Conseguiste estudiar todo a tiempo, Y lo suficientemente riguroso, como para poder entenderlo?**

ADICIONES:

Con frecuencia, tiendo a ser el único que encuentra problemas serios en las demostraciones, las formulaciones de teoremas y los ejercicios elaborados en clase. O todos los demás entienden todo, la mayoría o no entienden y no les importan los posibles problemas. A menudo, encuentro agujeros en las pruebas y que faltan hipótesis en el teorema. Cuando se los presento al profesor, dice que tengo razón y menciona que soy muy preciso. ¿Cómo es esto preciso, cuando el teorema no se cumple en su estado actual? ¿Se supone que debemos entender las pruebas? ¿Las pruebas son en realidad solo bocetos? ¿Cómo diablos se supone que uno puede descubrir verdades matemáticas? ¿Es el estudio de las matemáticas solo una gran broma y se supone que no debes tomártelo demasiado en serio?

NOTA:

Tengo un montón de deportes que me gustan y solía hacer. Además, antes tenía una vida social perfectamente buena, así que no es necesario que me des consejos al respecto. No socializo ni practico deporte porque digerir las pruebas y tratar de entender las ideas detrás de todo esto consume todo mi tiempo. Si voy de excursión, me quitará 2 días, uno para caminar + uno para descansar y regenerarse. Si voy a entrenar MMA, no estaré concentrado todo el día. No puedo simplemente cambiar del boxeo a la persecución de diagramas en un momento. Además, no puedo estudiar solo durante media hora. La forma en que estudio es: abro el libro, busco lo que ya sé pero olvidé del día anterior, y luego voy de teorema en teorema, de prueba en prueba, corrigiendo errores, agregando aclaraciones, etc. etc. Para agregar , tengo la mala costumbre de tener dificultad para iniciar las cosas. Sin embargo, cuando arranco, arranco 'mi motor' y tengo dificultad para detener, especialmente si va bien. Es por eso que, sin querer, paso una o dos horas antes de estudiar haciendo las cosas más irrelevantes, solo para evitar estudiar. Esto sucede especialmente cuando tenía más matemáticas de las que podía meter en mi garganta que tengo, para prepararme mentalmente para comenzar a estudiar. Pero, como mi motor realmente arranca y el estudio va bien (probé mucho, entendí mucho), es difícil para mí parar, así que a menudo me quedo tarde en la noche, hasta las 4 am, 5 am y 6 am Cuando el día de llega el examen, no me duermo nada, y la noche y el día se invierten . Me acuesto a las 13h y me despierto a las 21h... Sé que no es bueno pero parece que no puedo romper este hábito. Si soy inútil durante todo el día,. Sé que esto no debería suceder si uno ama las matemáticas. Sin embargo, cuando te 'imponen' qué, cuánto y en qué cantidad de tiempo tienes que estudiar, empiezas a desanimarte por las matemáticas. Las matemáticas dejan de ser un placer/diversión y se convierten en un trabajo duro que solo hay que hacer.ingrese la descripción de la imagen aquí

Una observación de Gian-Carlo Rota es pertinente: "Los hechos de las matemáticas se verifican y presentan mediante el método axiomático. Sin embargo, hay que cuidarse de no confundir la presentación de las matemáticas con el contenido de las matemáticas. Una presentación axiomática de un hecho matemático difiere del hecho de que se presenta como una medicina diferente de la comida. Es cierto que esta medicina en particular es necesaria para mantener a los matemáticos a una distancia segura de los autoengaños de la mente. Sin embargo, entender las matemáticas significa poder olvidar la medicina y Disfruta la comida."
Lo anterior es un extracto de la p.96 de Matemáticas Indiscretas de Rota .
@Bill: Pensamientos indiscretos.
En cuanto a "tener un estilo de vida saludable", descubrí que definitivamente se aplica la Ley de Parkinson . Así como encuentro la necesidad de "hacer una cita con mi investigación" (dedicar una cierta cantidad de tiempo la mayoría de los días para pensar en mi investigación, esté progresando o no), también encuentro que si dejo de lado la tiempo para hacer ejercicio/comer fuera/etc., puedo usarlo para eso, pero si no lo dejo a un lado, no es que tenga mucho tiempo sin hacer nada. Aparta el tiempo.
Trataría de obtener una visión general de lo que estoy aprendiendo primero. Me gustaría pensar en ello como un gran lienzo; complete los detalles de acuerdo con lo que despierte su interés. Absolutamente positivamente no puedes aprender todo (ni siquiera cerca), pero puedes aprender de arriba hacia abajo en lugar de de abajo hacia arriba. Para sus comentarios sobre el rigor, puede encontrar esto interesante: cheng.staff.shef.ac.uk/morality/morality.pdf
Tenga en cuenta que la falta de sueño puede hacer que sea más difícil recordar cosas. Trata de dormir más (es decir, al menos 8 horas por noche), te hará más productivo e incluso podrías terminar ganando más tiempo del que inviertes en dormir por no olvidar tanto.
Esto realmente conmovió a la comunidad aquí; ¡Ninguna otra pregunta produce 6 respuestas largas en 6 horas! Así que @Leon: Valora el hecho de que no estás solo en esta situación hasta que encuentres un remedio. :)
Lo más probable es que la fuente de sus problemas sea la falta de sueño. Me resulta imposible ser eficiente sin estar bien descansado. (Es decir, necesito mis 9 horas) Además, piensa en los problemas de matemáticas justo antes de acostarte, de esa manera podrías soñar con ellos y parte de tu mente seguirá trabajando. Apenas la semana pasada, mientras estaba acostado en la cama tratando de conciliar el sueño, se me ocurrió una idea para resolver un problema en el que había estado pensando durante unas 6 horas ese mismo día.
Encontré este artículo Una guía de supervivencia para matemáticos de Peter G. Casazza "pensada como una guía de supervivencia para aquellos estudiantes, profesores y matemáticos que tienen problemas para interpretar la experiencia matemática". Tal vez te pueda ayudar.
Encuentro que salir a caminar me ayuda a pensar y aclarar las cosas en mi cabeza. Caminar me ayuda a resolver problemas. Además, no es exactamente correr una maratón, ¡pero dar muchos paseos me da suficiente ejercicio!
En cuanto a su memoria, una anécdota de Erdos podría tranquilizarlo: se olvidaba de las definiciones en el día a día y preguntaba, "¿qué es housdorff..." y luego se le explicaba y demostraba el teorema en cuestión. , al día siguiente alguien dice: "Vamos X sé un espacio housdorff" y Erdos pregunta "¿qué es housdorff...". No se trata solo de memorizar todo, sino más bien de internalizar las técnicas y comprender el panorama general. (Y siempre ayuda ser Erdos: P)
No recuerdo haber escrito esto, pero debo haberlo hecho porque me estás describiendo. Sufrí de estrés durante toda la universidad, algunas formas en las que me las arreglaba eran irme a dormir a medianoche sin importar la cantidad de tarea que tuviera (de lo contrario, soy un inútil) y caminar 15 minutos, incluso si era solo hasta mi automóvil. Empecé una práctica de meditación de 8 minutos al día, que pareció ayudar. Y trabajé en una biblioteca para poder hacer la tarea en el trabajo. Mi paso por la escuela perjudicó mi relación con mi novia, pero arreglamos las cosas y nos casamos después. Mi mayor consejo es que no te estreses con las calificaciones, ¿no?
"¿Cómo estudiar matemáticas... y tener... tiempo libre?" Solución: haz matemáticas en tu tiempo libre.
Haz matemáticas en tu tiempo libre y ten una orientación relajada, está bien si te pierdes algo. El trabajo duro simple no funcionará en matemáticas
El triángulo es bastante hermoso y veraz.
Tengo una idea, pero no estoy seguro de si ayudará. Ni siquiera sé si no empeorará las cosas, así que dejaré que tú juzgues si crees que podría empeorar las cosas. Una idea es leer las respuestas en math.stackexchange.com/questions/1333206/… . Otra idea es crear tu propio sistema de teoría numérica pura. Creo que todo el mundo puede seguir asimilando nueva información y reteniéndola a un ritmo distinto de cero, incluso si es un ritmo muy lento. Para entrenar su memoria, podría seguir descifrando más declaraciones demostrables en ese sistema
en tu cabeza. Creo que de vez en cuando descubrirás una declaración y habrás descubierto exactamente la misma declaración y la habrás olvidado tantas veces antes que finalmente la retendrás esta vez. Al no llevar un registro de lo que averigüe, puede mejorar lentamente en la retención de nuevas declaraciones que descubra y descubrir nuevas declaraciones que retendrá con un poco más de frecuencia que antes. Es posible que piense que no puedo descifrar ninguna declaración que sea realmente útil y se sienta atascado. Eso no importa. Todavía es mejor descubrir una declaración más y retenerla que no
más. Tal vez como las declaraciones que previamente descubrió que sentía que eran inútiles se pueden usar para calcular otras declaraciones, eventualmente descubrirá suficientes declaraciones para comprender tanto y luego se dará cuenta de que esas declaraciones no fueron inútiles después de todo porque eventualmente conducen a descubrimientos útiles . Si el sistema es lo suficientemente simple como para obtener el efecto contrario de que afirma haber obtenido una comprensión completa del sistema y parece tan simple, entonces podría extender el sistema a un sistema más fuerte para describir la declaración que todas las declaraciones que el sistema prueba son verdaderas.
Finalmente, mi tercera idea se basa en mi hipótesis incierta de que Math Overflow es adecuado para preguntas que generan una investigación matemática útil que no es demasiado difícil de hacer y es trasladar esta pregunta a MathOverflow para generar una investigación sobre cómo crear un curso adecuado para personas como usted. . Si no aceptan esa pregunta, entonces tal vez primero podría hacer una pregunta en Academia Stack Exchange para obtener más información sobre por qué enseñan de la manera en que lo hacen o por qué los cerebros de las personas aprenden de cierta manera, entonces tal vez tenga los recursos para preguntar. esa pregunta en MathOverflow. Si eres investigador, quizás también puedas
pregúntelo en ResearchGate. Si no, tal vez podría hacer una pregunta en Stack Exchange para obtener información que lo ayude a descubrir cómo podría convertirse en investigador, pero no sé qué sitio web de Stack Exchange.
También creo que probablemente sea mejor no obtener ninguna ayuda de un programa de computadora similar a Python para hacer cálculos para usted o buscar en Google para encontrar respuestas a las preguntas. Tal vez esas respuestas son respuestas que en realidad no necesitas porque siempre podrás descifrar mentalmente una declaración más y retenerla, pero usarlas te hará adquirir el hábito de no descifrar ninguna declaración por tu cuenta. Python no puede hacer el pensamiento creativo por usted y darle una declaración matemática verdadera equivalente a una respuesta escrita en inglés que le enseñará cómo ser realmente bueno en matemáticas. Puede haber un
declaración matemática comprobable de que una vez que la aprenda, podrá descubrir una técnica matemática específica muy artificialmente inteligente, pero probablemente sea más probable que la encuentre si no recibe ayuda de Python que si la obtiene. De acuerdo con matheducators.stackexchange.com/questions/7718/… , las personas que estudian libros avanzados a una edad temprana probablemente tengan menos probabilidades de resolver cosas por su cuenta y se queden atascados cuando tienen una pregunta que no pueden encontrar un libro o un resultado de búsqueda de Google que responda.

Respuestas (27)

En mi opinión, la pregunta central que debes hacerte es cuál es el objetivo final de tus estudios. Como ejemplo, la vida universitaria estadounidense, tal como se muestra en la película, es hedonista y ciertamente no se centra en los estudios reales. Su ejemplo es todo lo contrario: se describe a sí mismo como un asceta dedicado a la erudición.

Muchas personas consideran importante llevar una vida equilibrada . Si esa persona se enfrentara a su situación, podría buscar algún compromiso, por ejemplo, invertir menos tiempo en los estudios a cambio de calificaciones más bajas. Si las cosas no funcionan, podrían considerar optar por no participar en toda la empresa. Su punto de vista puede ser diferente: para usted, la dimensión más importante es el crecimiento intelectual y está dispuesto a sacrificarlo todo por ello.

Se ha mencionado en otra respuesta que llevar un estilo de vida saludable podría contribuir a sus estudios. Las personas tienden a "quemarse" si trabajan demasiado. He conocido a personas así, y tenían que "refrescarse" periódicamente en algún lugar lejano. Por el contrario, las actividades no curriculares pueden ser vigorizantes y refrescantes.

Otro aspecto similar es el de "estar ocupado". Algunas personas descubren que al realizar múltiples tareas se vuelven más productivas en cada uno de sus "frentes" individuales. Pero ese estilo de vida no es para todos.

Volviendo a mi punto original, ¿qué espera lograr al tener éxito en la escuela? ¿Estás apuntando a una carrera académica? ¿Carrera profesional? En América del Norte, la educación superior se ha convertido en un rito de iniciación, que muchos graduados encuentran muy problemático por el costo que implica. Para ellos, el problema suele ser económico: la educación es costosa en América del Norte.

Es posible que descubras que, habiendo completado tus estudios, debes cambiar tu vida por un rumbo muy diferente. Es posible que llegue a darse cuenta de que ha desperdiciado algunos de los mejores años de su vida estudiando duro hasta el punto de excluir todo lo demás, un esfuerzo que eventualmente no lo llevará a ninguna parte. Este es el peor de los casos.

Más concretamente, le sugiero que planifique con anticipación y considere si el costo vale la pena. Eso requiere tanto una evaluación seria de su propio valor como algo de especulación sobre el futuro mercado laboral. También debe estimar qué tan importante va a considerar estos estudios actuales en su futuro, tanto desde la perspectiva económica como desde la "cultural".

Todo esto puede parecer desalentador, pero su situación, tal como la describe, es bastante miserable. No solo no está satisfecho con él, sino que también parece problemático para un observador externo. Sin embargo, sospecho que estás exagerando, viendo la situación desde una perspectiva romántica y heroica. Por lo tanto, es mejor hablar con personas que lo conozcan personalmente.

Aún mejor, hable con personas mayores que usted y en la siguiente etapa de la "vida". Tienen una perspectiva más amplia de su situación, que sus conocidos aún recuerdan vívidamente. Sin embargo, incluso sus recomendaciones deben tomarse con cautela, ya que sus preocupaciones actuales son solo una parte del panorama más amplio, la "vida" que lo abarca todo.


Finalmente, algunas palabras más pertinentes al tema que nos ocupa.

En primer lugar, la estrategia de aprendizaje. Creo que la mejor manera de aprender es resolver ejercicios desafiantes. El consejo dado aquí, tratar de "reconstruir" el libro de texto antes de leerlo, parece consumir mucho tiempo y, en mi opinión, concentrar el esfuerzo en el lugar equivocado.

Lo mismo ocurre con la memorización de teoremas: a veces uno solo puede "comprender" realmente la demostración de un teorema estudiando un tema más avanzado. Incluso el investigador que originalmente presentó la prueba probablemente no la entendió "realmente" hasta que se desarrolló una perspectiva más amplia.

Memorizar teoremas no es tu elección sino una necesidad. Siempre me disgustó la regurgitación y es lamentable que esto te sea forzado. Me alegro de que mi escuela nos dé problemas reales para resolver; de todos modos, eso está mucho más cerca de la investigación. Ya que tiene que pasar por este lamentable proceso, intente idear un método de memorización que también tenga otros beneficios, tal vez apunte a una mejor comprensión de "lo que está sucediendo" en lugar de los pasos reales en sí. Esta es una habilidad importante.

En segundo lugar, una de las respuestas sugiere tratar de deducir tantos teoremas como sea posible como lo "matemático" que se debe hacer después de ver una definición. Sugeriría más bien lo contrario: primero averigüe qué implica la definición y luego intente comprender por qué el concepto se definió en primer lugar y por qué de esa manera particular.

Es común en matemáticas comenzar a estudiar un tema con una larga lista de "definiciones importantes", que no tienen importancia en absoluto en esa etapa. Habrás entendido el tema cuando puedas explicar de dónde vienen estas definiciones, qué objetos describen; y cuando puedes "sentir" estos objetos intuitivamente. Esto está muy lejos de poder deducir algunos hechos que se derivan más o menos directamente de las definiciones.

@Yuval: Muy bien escrito (incluso tuve que buscar algunas palabras), y sobre todo muy buenos consejos, estoy muy agradecido por tus sugerencias y tu tiempo. Para mí, el tema ciertamente no es económico, ya que los estudios aquí son gratuitos. "Siempre me disgustó la regurgitación y es lamentable que esto te sea forzado". No, no, ciertamente no estamos obligados a memorizar cosas como en las ciencias sociales de ninguna manera, pero con cada nuevo tema, se supone que conocemos la mayor parte del material de los anteriores, que es mucho, y constantemente actualizo y olvido. .
Es realmente frustrante que entiendo un tema y en 6 meses casi se me ha ido de la cabeza. Más aún, cuando algunos compañeros de clase parecen tener mucha mejor memoria. Por supuesto, cuando vuelvo a aprender, va más suave y rápido, pero el punto es que tengo que volver a aprender una y otra vez. Y dado que la cantidad de material solo aumenta, esto se vuelve cada vez más difícil. Hay muchos campos de las matemáticas, y el número de todos los teoremas, lemas, proposiciones y definiciones es inmenso. Por supuesto, la intuición siempre ayuda, pero no crea milagros. Como en los deportes, sin cardio, un atleta está condenado.
@Yuval: Por cierto, casi no noté que había otra (tu) respuesta. Me dijeron que usar @ informa a una persona de otro comentario dirigido a él. ¿Dónde/cómo noto esto, sin revisar todas mis preguntas cada vez?
@Leon: Hacer investigación matemática implica acumular una gran cantidad de conocimiento y aprovecharlo continuamente; no necesariamente recurriendo a un lema oscuro o incluso a un teorema moderadamente importante, sino más bien a través de analogías y mediante una comprensión intuitiva de la naturaleza de algunos objetos matemáticos y técnicas para razonar sobre ellos. Si honestamente descubre que sigue olvidando el material antiguo, y con eso me refiero a lo esencial, no a los resultados avanzados, probablemente esté estudiando demasiado rápido, demasiado o con demasiado nerviosismo.
En cuanto a su pregunta técnica, creo que recibe una notificación: aparece una señal especial en la parte superior de la página. Y si marca la casilla correcta al hacer la pregunta, se le notificará por correo electrónico cada vez que alguien responda a su pregunta.
Creo que en su mayoría no olvido lo esencial, pero para aplicar cierto lema, uno tiene que conocer las hipótesis. Hay muchas trampas, muchos contraejemplos, en topología, álgebra y análisis que demuestran claramente lo fácil que es llegar a una conclusión incorrecta.
Lleva tiempo acostumbrarse a una nueva forma de conocimiento, especialmente a una tan complicada como las matemáticas. En algún momento alcanzarás un nuevo nivel de comprensión; de repente, todo el material anterior, que solía parecer tan difícil, se volverá manifiestamente simple. Comenzará a recordar conceptos y resultados básicos poco después de resolver algunos ejercicios relevantes. Puede parecer incrédulo, pero si perseveras, eventualmente sucederá. Sea paciente y relajado. Todos estuvieron alguna vez en tu situación actual. Las cosas mejoran eventualmente.
Puede que esta no sea la respuesta mejor calificada (todavía), pero sin duda es la más completa en mi opinión. Estuve de acuerdo con la observación de que algunos de los otros métodos descritos en esta página requieren mucho tiempo. Los probé, y terminas aprendiendo mucho menos que si hubieras, digamos, solo cubierto algunos materiales básicos y probado algunos ejercicios, aumentando gradualmente la dificultad cada vez.
¿Qué hizo @YuvalFilmus entre 2002 (fin de maestría) y 2009 (comienzo de doctorado)?

Déjame decirte que lo único que he estado haciendo durante los últimos cuatro años de mi vida son las matemáticas. He disfrutado mucho de la experiencia, pero también he tenido momentos en los que no estaba seguro de cómo abordar mi aprendizaje. Creo que no hay una regla que funcione para todos; sin embargo, permítanme responder algunas de sus preguntas. Espero poder ayudar:

Pregunta: ¿Cómo estudiar matemáticas de la manera correcta?

Respuesta : Creo que la mejor manera de estudiar matemáticas es la siguiente. Supongamos que ya ha elegido un libro de matemáticas sobre un tema que realmente le interesa aprender. Cuando lea el libro, intente pensar activamente sobre el tema de diferentes maneras . Por ejemplo, si se presenta una definición, dedique al menos 30 minutos a pensar en la definición. Si está estudiando un libro sobre álgebra lineal y se presenta la definición de un "operador nilpotente", debe intentar descubrir algunas propiedades básicas sobre los operadores nilpotentes por su cuenta sin seguir leyendo. Esto puede ser difícil al principio, pero en última instancia, la capacidad de hacerlo de manera efectiva con tantas definiciones como sea posible es importante en la investigación matemática.

Tomemos el siguiente ejemplo en la teoría elemental de grupos. El autor presenta la definición de un subgrupo máximo de un grupo finito GRAMO : un subgrupo METRO de GRAMO se dice que es un subgrupo maximal si METRO es un subgrupo propio de GRAMO y si no hay subgrupos propios de GRAMO que contiene estrictamente METRO . Debes tratar de seguir los siguientes pasos:

(1) ¡ Encuentre ejemplos de subgrupos máximos en grupos finitos y comience con los ejemplos más triviales ! Por ejemplo, el grupo trivial no puede tener un subgrupo máximo. Si entiendes esto, has captado un punto de la definición. El siguiente paso es considerar los grupos cíclicos más simples. ¿Cuáles son los subgrupos máximos del grupo cíclico de orden 2? ¿Cuáles son los subgrupos máximos del grupo cíclico de orden 4? Piensa en ejemplos básicos como este. Cuando esté listo, intente formular un teorema general por su cuenta que concierna a los subgrupos máximos de un grupo cíclico de orden norte . Debes llegar al teorema de que un subgrupo H de un grupo cíclico GRAMO es máxima si y solo si el número | GRAMO | | H | es primo

Continúe buscando otros ejemplos de subgrupos máximos en un grupo finito. El siguiente paso es considerar el grupo de 4 de Klein y los grupos de permutación de orden bajo. Espero que en este punto estés realmente fascinado por el concepto de un subgrupo máximo. Al principio, la definición puede parecer algo arbitraria; sin embargo, ahora que lo ha pensado, ha comenzado a obtener un sentido de "propiedad" sobre la definición .

(2) Ahora es el momento de formular y probar algunos teoremas sobre subgrupos máximos . Una vez más, piense en los ejemplos más fáciles . Una cosa que puede ser desalentadora para un principiante es no poder responder una pregunta que parece fácil durante un largo período de tiempo. ¿Cuál es un buen ejemplo de un teorema fácil? Puede estudiar aquellos grupos finitos que tienen exactamente un subgrupo máximo. ¿Qué se puede deducir acerca de tal grupo? Si encuentra que está atascado, intente volver a los ejemplos de subgrupos máximos que ideó anteriormente. De hecho, esta pregunta puede responderse de manera bastante satisfactoria; un grupo finito con un único subgrupo máximo es cíclico de primer orden de potencia.

(3) El siguiente paso es conjeturar algunas propiedades más sobre los subgrupos máximos basados ​​en los ejemplos que ideó en (1) . Por ejemplo, averiguaste que si H es un subgrupo máximo de un grupo cíclico finito GRAMO , entonces | GRAMO | | H | es un número primo. ¿Es esto cierto para todos los grupos? GRAMO ? ¿Puedes pensar en grupos? GRAMO para el cual esto es cierto?

Observe cómo se puede deconstruir una definición simple para llegar a una serie de preguntas interesantes . Esto es lo que hace un matemático todo el tiempo y es una habilidad muy importante. Puede parecer difícil al principio, pero hacer esto hará que las matemáticas sean aún más emocionantes y te dará un sentido de "propiedad" sobre el contenido. Usted resolvió esta pieza de matemáticas . Esta es la forma en que aprendo matemáticas y puedo decirte con confianza que si practicas esto, pronto se convertirá en la norma.

¿Qué haces después de mirar la definición y haber pensado en ella extensamente? Continúas leyendo el texto. Es muy probable que note que el autor menciona algunos de los resultados que descubrió por su cuenta. Con suerte, habrá resultados que el autor no ha declarado. Si es así, puede ser buena idea preguntar (en esta web, por ejemplo) sobre la originalidad del resultado.

Sin embargo, encontrará teoremas relacionados con las definiciones en los que simplemente no pensó. Debes resistir la tentación de ver las demostraciones de estos teoremas y más bien debes tratar de demostrar estos teoremas por tu cuenta . Piensa en el teorema durante al menos unas horas antes de darte por vencido. Tenga en cuenta que los teoremas con demostraciones bastante breves pueden requerir ideas muy originales y, por lo tanto, no debe presionarse para probar el teorema en poco tiempo.

Al principio, te tomará mucho tiempo probar algunos teoremas. Habrá teoremas de rutina y estos deben probarse con bastante rapidez. Pero también habrá teoremas difíciles. A medida que adquiera experiencia, su pensamiento será más rápido y estos teoremas le resultarán más fáciles . Sin embargo, no debe esperar que este sea el caso inicialmente.

Por ejemplo, podría encontrar el siguiente teorema en álgebra lineal: si norte es una transformación lineal nilpotente de un espacio vectorial V a sí mismo y si la dimensión de V es norte , entonces norte norte = 0 . Averiguar cómo probar este teorema por su cuenta es una experiencia muy valiosa y gratificante . Si aún no lo has visto, te sugiero que intentes probarlo. Sin embargo, no es demasiado difícil.

Pregunta: ¿Cómo evitar olvidar las matemáticas?

Respuesta : También solía olvidarme de las matemáticas cuando las aprendí. He hablado con varios matemáticos sobre esto y me han dicho exactamente lo mismo. El punto es que solo tienes que aceptar desde el principio que olvidarás lo que aprendas. Sin embargo, hay maneras de asegurarse de mantener esto al mínimo.

Por ejemplo, la mejor manera de no preocuparse demasiado por olvidarse de las matemáticas es resolver las matemáticas por su cuenta . Por ejemplo, considere los pasos que sugerí en la pregunta anterior. Incluso si haces esto, aún puedes olvidarte de las matemáticas, especialmente si el resultado en cuestión fue bastante fácil de probar. (Tenga en cuenta, sin embargo, que si el resultado es difícil de probar y dedica, supongamos, 10 horas para probarlo, entonces probablemente nunca lo olvidará por el resto de su vida ).

El mejor método a seguir es escribir todas las matemáticas que aprendes . Tome notas copiosas. Por ejemplo, cuando leí "Análisis real y complejo" de Walter Rudin el año pasado, eliminé 3 libros completos de notas. De hecho, ¡escribí 600 páginas de matemáticas cuando solo leí 315 páginas!

Escribe cada definición, cada teorema y cada demostración . Las definiciones y los teoremas se deben producir palabra por palabra del libro, ya que es importante asegurarse de que su comprensión del rigor sea correcta. Sin embargo, las demostraciones deben escribirse con sus propias palabras.

Pregunta: ¿Cómo tener un estilo de vida saludable?

Respuesta : Me temo que realmente no tengo una buena respuesta para esto. En los cuatro años que llevo estudiando matemáticas, ciertamente no he hecho nada más. Por lo tanto, realmente no puedo dar consejos sobre cómo administrar el tiempo. Si eres un estudiante serio de matemáticas, pasarás prácticamente todo el día estudiando la materia. Esto es inevitable. Por ejemplo, me propongo objetivos todos los días de cuántas matemáticas deseo hacer y, por lo general, termino haciendo matemáticas sin parar. Sin embargo, realmente disfruto esto y no desearía tenerlo de otra manera.

Pero puedo ofrecerte un pequeño consejo: trata de levantarte temprano , supongamos, a las 6:00 AM. Sin embargo, asegúrese de dormir al menos 8 horas; por lo tanto, acuéstese a las 9:00 p. m. Dormir es uno de los puntos más importantes a la hora de estudiar. Durante muchos años de hacer matemáticas, descubrí que soy más productivo y enérgico antes de las 12:00 . Si puede terminar la mayor parte de su trabajo antes de las 12:00, entonces estará en una muy buena posición para hacerlo bien cada día. Además, trate de evitar comer comidas copiosas . Las comidas copiosas a menudo hacen que pierdas la concentración y esto, a su vez, puede llevarte a perder varias horas.

Creo que el punto más importante cuando te propones alcanzar cualquier meta en tu vida es tomarla día a día, hora a hora, incluso minuto a minuto . A menudo, puede complicar demasiado las metas al pensar en lo que le gustaría hacer durante el próximo año o incluso un mes. Si trabaja duro todos los días y establece metas realistas, todo debería ser posible .

¡Espero haber ayudado! (Espero que mi uso del texto en negrita no se considere ofensivo; simplemente lo usé para resaltar algunos de los puntos clave de mi respuesta).

Descargo de responsabilidad (25 de diciembre de 2013): esta respuesta fue escrita cuando tenía 16 años y no representa necesariamente mi visión actual de las matemáticas. (Algunos puntos, por ejemplo, "escribe todas las matemáticas que aprendas" no es algo que recomendaría a nadie hoy en día). Pero dejo mi respuesta aquí porque creo que es un consejo razonable en general y claramente ha sido útil para muchas personas como se evidencia por los 77 votos a favor.

El texto en negrita siempre es bienvenido, para distinguir partes importantes. He pasado por la fase de aprender a leer textos matemáticos con suficiente rigor y de hacerme preguntas. El problema es que me hago demasiadas preguntas y dedico demasiado tiempo a entender las cosas. Estoy preguntando cómo aprender matemáticas de forma rápida Y rigurosa.
"El mejor método a seguir es escribir todas las matemáticas que aprendes. (...) Escribe cada definición, cada teorema y cada prueba". Esto ha pasado por mi mente varias veces, y lo he intentado. Estoy de acuerdo en que esta es una de las mejores maneras de dominar y memorizar el material, siempre que las cosas que escriba estén en sus propias palabras y pensamientos. Pero claro, el gran problema es, de nuevo: el tiempo.
@wildildildlife No estoy de acuerdo con esto hasta cierto punto. Es cierto que escribir todas las matemáticas que aprendes lleva tiempo, pero me parece que lleva aún más tiempo volver a aprender las matemáticas que olvidaste y no escribiste. Por ejemplo, aprendí la teoría de grupos finitos tres veces porque olvidé el material cada vez. (Por supuesto, lleva menos tiempo "volver a aprender" el material porque ya lo ha visto, pero la cantidad total de tiempo que lleva cimentar el material en su memoria a largo plazo parece mucho más que la cantidad total de tiempo que lleva escribir las matemáticas.)
Bueno, todavía estoy de acuerdo contigo. Pero es un hecho que durante mis cursos no tengo tiempo para hacer esto. Hacer todos los ejercicios que tengo que hacer, y leer para poder hacerlos, ocupa casi todo el tiempo. El resto se dedica a tratar de construir y leer pruebas. Escribir todo esto de una manera comprensible requeriría 3 días adicionales a la semana. Tal vez los mensajes de texto en vivo (durante las conferencias) ayudarían. De lo contrario, tendría que hacerlo durante mis preciosas vacaciones de verano.
O tal vez deberíamos terminar math.stachexhange, eso liberaría algo de tiempo :)
Creo que las matemáticas no deberían aprenderse como dices 'aprendiste la teoría de grupos finitos tres veces'. En la investigación es mejor saber dónde encontrar los resultados que conocer todos los resultados. Además, creo que tratar de probar cada teorema que encuentras por ti mismo es un poco exagerado. Nadie puede volver a probar todo lo que se ha escrito antes. Pruebe con Evans y Gariepy, Teoría de la medida y propiedades finas de las funciones; la mayoría de las pruebas tienen páginas. Claro, en algunos momentos después de mucha lectura, las pruebas vienen naturalmente, pero no en la primera lectura.
@BeniBogosel Estoy de acuerdo contigo. Entiendo que mantener todos los resultados matemáticos en cierta colección de libros de texto en la mente de uno es bastante inútil dado que incluso eso será solo una pequeña proporción del conocimiento matemático actual y eso, por sí solo, no ayudará a uno a hacer algo que alguien nunca ha hecho. antes. Además, incluso si uno no prueba cada teorema por su cuenta, aún puede dividir cada teorema en pasos (o párrafos) y probar cada paso (o párrafo) por su cuenta. Creo que esto es más eficiente y también ayuda a entender muy bien la prueba.
"... la mejor manera de no preocuparse demasiado por olvidarse de las matemáticas es resolver las matemáticas por su cuenta". Definitivamente. Es difícil recordar la vista desde la cima de una montaña si solo la viste en una foto; es fácil si subiste a la montaña para verlo.
@AmiteshDatta, cuando dices "¡De hecho, escribí 600 páginas de matemáticas cuando solo leí 315 páginas!", ¿No perdería más tiempo releer sus notas que releer el libro en sí?
Hola @Pacerier, gracias por tu comentario. En realidad, nunca volví a leer mis notas (aunque tendría curiosidad por hacerlo en algún momento, aunque solo fuera para ver lo que estaba pensando la primera vez que estaba aprendiendo el tema). Si reviso un tema, entonces, por la razón que mencionas, volvería y volvería a leer las partes relevantes del libro (lo que probablemente sería una mejor exposición que mis notas preliminares, de todos modos). Hoy en día, no adopto este estilo de aprendizaje; lleva demasiado tiempo y no necesariamente me ayuda a recordar/comprender mejor el tema.
@AmiteshDatta, ¿todavía conservas esas 600 páginas de notas? ¿Son copias electrónicas?
Hola @Pacerier, las escribí en varios cuadernos a mano y probablemente estén en algún lugar de mi casa. ¡Quizás trate de encontrarlos ahora que lo mencionas!
@AmiteshDatta Estoy empezando a aprender Real Analysis por mi cuenta. Seguiré su consejo como pensar en la definición durante media hora, deducir propiedades básicas, demostrar el teorema sin ver su prueba. ¿Sin embargo, cree que este enfoque es para un principiante promedio? Gracias
Hola @JPG, ¡gracias por tu comentario! Es difícil para mí recomendar algo sin conocer tu situación específica, pero creo que lo principal es pensar en las matemáticas a tu manera, además de lo que estás aprendiendo. Por supuesto, hay muchas maneras de hacer esto y lo que sugerí es solo una. Vale la pena probar varios enfoques y luego decidir cuáles funcionan para usted.
@AmiteshDatta, pero el tiempo es un recurso limitado. Creo que lo que intenta es ahorrar tiempo aprendiendo de tus errores. Él está preguntando cuáles son las cosas que has hecho que has encontrado que son una pérdida de tiempo. Básicamente, ¿cuáles son los errores que has cometido?
Hola @Pacerier, gracias por tu comentario; ese es un buen punto. Creo que es justo decir que no fue el mejor enfoque para mí tomar tantas notas, en retrospectiva. Realmente no me ayudó a recordar mejor las matemáticas; Creo que es mejor no preocuparse demasiado por olvidar las cosas y simplemente estar en paz con ellas. A medida que uno adquiere más experiencia matemática, las ideas realmente importantes serán claras, así como una visión (más) amplia de lo que tratan ciertos aspectos del tema (a veces es mejor olvidar los detalles de las demostraciones, etc.)...
Además, probar resultados por su cuenta definitivamente ayuda a involucrarse activamente en el tema, y ​​es una excelente manera (creo) de evaluar la comprensión de uno desde el principio (fue un pequeño salto para mí cuando comencé a leer libros de texto de matemáticas serios después de cálculo , porque las soluciones de los ejercicios no estaban presentes, por eso inicialmente comencé a tomar los teoremas como ejercicios con soluciones). Sin embargo, a la larga, puede llevar mucho tiempo probar todo (o incluso muchas cosas no triviales) por uno mismo; es un equilibrio complicado. (Además, algunas cosas parecen triviales si tienes la perspectiva correcta).
En última instancia, la forma en que uno aprende matemáticas realmente depende de muchos factores, como la experiencia, la madurez matemática, la cantidad de tiempo que se tiene, la forma en que uno puede aprender cosas nuevas, etc., por lo que es difícil dar una respuesta realmente específica. . Me refiero a que una cosa que funcionó bien para mí al principio no fue necesariamente aprender las cosas en orden, sino más bien como las veía interesantes. Por ejemplo, estudié topología (topología algebraica básica y de conjuntos de puntos) sin haber aprendido realmente álgebra lineal, análisis real y análisis complejo; Simplemente busqué algunas cosas en un análisis real cuando las necesitaba...
A la larga, me las arreglé para cubrir los temas básicos, y esto me convenía más que trabajar "sistemáticamente" en las matemáticas. En principio, no recomendaría esto a un estudiante principiante de matemáticas (de hecho, puede que ni siquiera se lo recomiende a una versión más joven de mí mismo), pero así es como aprendí, al menos antes. La sugerencia principal que honestamente puedo dar es ser lo más creativo posible y encontrar su propio camino a través de las matemáticas, ¡y luego dar consejos a otras personas en función de lo que ha aprendido! A veces, la única forma de aprender de algunos errores es cometerlos uno mismo.
Hola, @JPG, consulta mis cuatro comentarios anteriores en los que proporciono una respuesta más elaborada a tu pregunta.
@AmiteshDatta Hmmm, gracias
@AmiteshDatta ¿Entonces tenías 16 años y hacías teoría de grupos? Hombre...
Hola, @AmiteshDatta, estoy viendo esta publicación ahora. Cuando dijiste "tómalo día por día, hora por hora..." en las últimas líneas, ¿significa eso que hiciste matemáticas simplemente despertándote y enfocándote en tratar de entender algún concepto, por ejemplo, y despertaste uno? día y notó que terminó 10 o más temas avanzados? Pregunto porque a menudo siento que puedo entender en lo que estoy trabajando actualmente, pero ver el beneficio a largo plazo es difícil de entender. Es por eso que me parece interesante que parezca estar diciendo que nunca se preocupó por el beneficio/ganancia a largo plazo. ¡Gracias!
Hola @johnfowles, ¡gracias por tu comentario! Sí, supongo que me enfoqué en algún objetivo concreto cada día que pensé que era alcanzable (pero solo esforzándome). Creo que una de las cosas que me ayudaron a motivarme fue tener una meta a largo plazo en mente (algún gran proyecto, por ejemplo, cuando estaba aprendiendo, tratando de leer todo un libro de matemáticas) y elaborar un plan diario para ayúdame a lograr ese objetivo. Sabía que si me perdía un día, me sería muy difícil ponerme al día, por lo que me obligó a trabajar duro todos los días y siempre tenía en mente una meta a largo plazo.
"Supongamos que a las 6:00 a. m. Sin embargo, asegúrese de dormir al menos 8 horas; por lo tanto, acuéstese a las 9:00 p. m.". 9-6 son 9 horas ;)
@AmiteshDatta ¿Aprendiste teoría de grupos a los 16 años? !!!

A la mayoría de las personas probablemente no les gustará esta respuesta, pero las matemáticas son un campo en el que existe una separación inestable entre aquellos con una comprensión del calibre de un genio y aquellos que solo pueden sobrevivir a través del trabajo duro y obstinado. Demasiadas personas quieren hacer pruebas y matemáticas ingeniosas estéticamente agradables para una carrera que se encuentra en la categoría de trabajo duro obstinado. Hablo como alguien que ha trabajado hasta la muerte durante los últimos 3 años para obtener un doctorado de la Ivy League. programa en matematicas aplicadas. Al lado de mis compañeros, la única ventaja que tengo es que puedo trabajar mucho más duro y, hasta cierto punto, soy mucho mejor escribiendo software. En términos de destreza matemática, todos me dominan.

Si, como admite, usted es promedio y tiene mala memoria (aparte de su tolerancia obviamente superior al promedio para el trabajo técnico difícil), entonces debe considerar que una carrera real en matemáticas puras no es adecuada para usted. Quiero tener cuidado para evitar la optimización de otros, así que tome mi consejo con pinzas. Puede que no sea adecuado para usted, y seguramente todos los demás comentaristas también tienen consejos perspicaces. Pero una cosa que creo que nunca funcionará para usted es simplemente "tratar de hacer ejercicio, comer bien y tener una vida equilibrada". Digan lo que digan los demás, esto no le sucederá a un matemático puro que tiene muy buen gusto en la belleza estética de los resultados, a menos que ese matemático realmente esté al nivel de un genio.

Tienes un suministro de talento limitado y un presupuesto de tiempo limitado. Es casi seguro que su pronóstico personal de que disfrutará de una carrera en matemáticas abstractas es incorrecto; pareces subestimar cosas importantes como el salario, la competitividad por la titularidad, las preferencias geográficas, etc.

Por ejemplo, tengo un amigo cercano que estudió aspectos muy puros de la teoría de números criptográficos. Hizo dos posdoctorados y no ganó prácticamente nada de dinero, sacrificó las relaciones personales para tratar de obtener puestos de profesor titular y, finalmente, no encontró trabajo en matemáticas puras. Aceptó un trabajo como programador para una empresa que fabrica software criptográfico. Pensó que al menos parte de su tiempo se dedicaría a investigar nuevas ideas abstractas en criptografía, pero resultó que no era cierto. En cambio, escribe programas Java la mayor parte del tiempo, aprende sobre nuevas investigaciones criptográficas aplicadas y escribe muy poco (aunque todavía incursiona en la investigación en su tiempo libre y, en mi opinión, está mucho más educado en investigación criptográfica que la mayoría de las personas que actualmente publicar en ese campo).

¿Es infeliz en esta situación? ¡No! En realidad, descubrió que para hacer un diseño de software adecuado, prácticamente todo se trata de comprender la abstracción correcta, la encapsulación de datos correcta, el patrón de diseño correcto, y esto no solo tiene un gran valor estético matemático, sino que también ofrece un mejor producto a un cliente. . Después de aclimatarse al desarrollo de software profesional, ahora ve todo tipo de paralelismos entre su trabajo anterior con ideas matemáticas abstractas y su trabajo actual con soluciones de software abstractas. Su conjunto de habilidades ahora tiene una demanda económica mucho mayor, no está presionado para competir por puestos permanentes y puede mantener un equilibrio muy saludable entre el trabajo y la vida personal gracias al horario de trabajo regular de su empresa.

Diría que, al igual que muchas pequeñas empresas fracasan, demasiados estudiantes de posgrado de ojos brillantes se ven a sí mismos como el próximo Godel, entusiasmado por puestos permanentes y "viviendo la vida de la mente". Son especialmente propensos a hacer lo que estás haciendo y a dejar que el resto de sus vidas se deteriore con la esperanza de poder perseguir lo que actualmente (probablemente erróneamente) creen que es su propia preferencia por la belleza abstracta que solo puede ser saciada (también una error) por matemáticas generalizadas. Muchas más de estas personas deberían admitir el hecho de que no tienen el talento suficiente y que las universidades que tienen que dedicar una cantidad cada vez menor de recursos para contratar a los mejores profesores titulares no deberían contratarlos.

Aquí hay algunos enlaces para considerar:

La economía también importa. La tenencia está disminuyendo considerablemente en muchas partes de la academia, y los profesores de matemáticas son especialmente notorios porque las matemáticas puras no generan subvenciones como lo hacen los proyectos aplicados. Con la llegada de los cursos en línea y los cursos abiertos, y sitios como Stack Exchange, la necesidad de profesores de matemáticas altamente especializados está disminuyendo a nivel universitario. Debe esperar que la competencia por los puestos de trabajo fijos aumente, y que si quiere un trabajo fijo tendrá que ir a cualquier lugar que los ofrezca, incluso si se trata de una pequeña universidad regional que no está cerca de ninguna ciudad importante, no tiene un ambiente cultural real. , y no atrae a estudiantes dotados. Sería un gran error no tener esto en cuenta.

Mi consejo para ti es este. Piensa bien qué es lo que te gusta específicamente de las matemáticas. Si le gustan las abstracciones y el pensamiento geométrico que a menudo son parte del análisis avanzado y la topología, entonces hay muchas rutas profesionales de matemáticas aplicadas/física aplicada/ingeniería que le ofrecerán la oportunidad de explorar preguntas matemáticas, pero también pondrán en práctica esa capacidad de abstracción geométrica. para trabajar escribiendo software para resolver problemas reales. Tu familiaridad con las matemáticas puras puede darte una ventaja profesional si te cambias a un campo como este. Si eso es lo que desea, podría estar en condiciones de competir de manera más efectiva por subvenciones y trabajos docentes, y en la medida en que domine las habilidades de programación, tendrá habilidades comercializables para obtener diferentes trabajos si surge la necesidad.

Si prefiere el pensamiento más abstracto que a menudo acompaña al álgebra y la teoría de números (es decir, si es un tipo de matemático puro que "soluciona problemas" según la definición de Timothy Gowers , entonces creo que encontrará mucho para disfrutar sobre el diseño de software Es posible que le resulte más útil centrarse en problemas abstractos en informática e ingeniería de software.

Si lee un buen libro de historia de las matemáticas (por ejemplo, Stillwell), notará que (a) la mayoría de las buenas matemáticas abstractas comienzan siendo una especie de intuición ad hoc, "probablemente sea cierto pero no puedo ver los detalles" de todos modos y solo se refina más tarde; y (b) las cosas más increíbles inventadas en matemáticas no fueron inventadas por personas que pensaron que las matemáticas eran la forma en que necesitaban ganarse la vida. La gente se ha vuelto loca por resolver problemas matemáticos durante milenios, se ha quedado despierta hasta altas horas de la noche, ha llevado vidas románticas destructivas y ha perdido salud. Si realmente amas las matemáticas, nunca serás feliz haciéndolas a medias como lo requiere un equilibrio saludable entre el trabajo y la vida, y muy pocas personas son realmente capaces de mantener una carrera como esa. La mayoría finalmente deja de esforzarse en la parte matemática y se siente insatisfecho con su carrera.

Ganarse la vida siendo un matemático puro es un concepto muy moderno que surgió en gran parte debido a las implicaciones de la integración de Lebesgue en el análisis y la teoría de la computabilidad en las ciencias de la computación. Y ahora que tenemos suficiente control sobre esos campos y sus subsiguientes hijos, simplemente no hay suficiente material para apoyar a muchos matemáticos de carrera. Es casi seguro que los avances matemáticos significativos en los próximos 50 años provendrán de aficionados muy inteligentes y dedicados, que resuelven problemas en lugares como Stack Exchange o Polymath .

Y no hay ninguna razón por la que no pueda encontrar algunos problemas de nicho en los que le guste trabajar, hágalo en su tiempo libre y, mientras tanto, tenga una carrera satisfactoria y económicamente sensata que le brinde una vida más cómoda. Por muy inteligente que seas, no sería prudente dejar de considerar este tipo de cosas en tu juventud. Muchos más estudiantes de matemáticas deberían hacerlo también.

De hecho, la falta de financiación realmente atroz e injusta de los estudiantes y la inflación de los puestos de postdoctorado se debe en gran medida a que los jóvenes ingenuos que piensan que obtendrán automáticamente la titularidad si se esfuerzan, y que piensan que el amor nerd por la ciencia estética es algo bueno para basar una elección de carrera en, parecen aceptar sin cuestionar posiciones académicas mal pagadas y con seguro insuficiente sin duda alguna. Confía en mí, no quieres ser simplemente otra de esas personas.

Este es el único buen consejo aquí.
Parece estar basado en la experiencia personal.
Algo de eso es. Pero los artículos vinculados sobre la falta de una prima salarial para los doctores en ciencias de la computación y matemáticas sobre los de maestría, y también el aumento de posdoctorados como investigadores principales (por lo tanto, más concentración de empleo académico en los niveles más bajos y menos en asociados). o puestos de profesor titular) provienen de fuentes externas que se derivan de investigaciones revisadas por pares. Otras observaciones, como tener un presupuesto de tiempo limitado y la necesidad de hacer algo económicamente productivo, son atributos más o menos predeterminados de la naturaleza. Entonces, si a una persona le importan estos, es menos anecdótico.
El mundo necesita más matemáticos, que trabajen en campos distintos de las matemáticas.
Como un programador que está aprendiendo matemáticas. No estoy seguro de cómo me siento acerca de esto.
¿Puedes elaborar?
"¿Está descontento en esta situación? ¡No! En realidad, descubrió que para hacer un diseño de software adecuado, prácticamente todo se trata de comprender la abstracción correcta, la encapsulación de datos correcta, el patrón de diseño correcto, y esto no solo tiene un gran valor estético matemático , pero también ofrece un mejor producto a un cliente" - no, esto simplemente no es cierto. El "diseño de software" no se acerca en absoluto a la investigación matemática y está mucho más cerca de un trabajo manual repetitivo. Uno puede disfrutarlo, como un buen carpintero disfruta haciendo muebles bonitos, pero no es un verdadero desafío intelectual.
@ Mr.F (en caso de que alguien venga a leer esto) Creo que sé lo suficiente sobre "ingeniería de software" para afirmar nuevamente mi opinión de que no es un desafío intelectual que se acerque a la investigación matemática.
@ Mr.F: En mi opinión, John Donn tiene razón en el sentido de que lo que dice describe con precisión la ingeniería de software en general. Siempre habrá excepciones en las que la pureza y la elegancia sean muy apreciadas, pero la mayoría de las veces la ingeniería de software está ahí para alimentar a los ojos de dólar. Muchas empresas conocidas entregan constantemente software lo antes posible para obtener el dinero a pesar de que ya se sabe que hay miles de errores presentes en él. La industria está impulsada casi en su totalidad por el dinero.
@ Mr.F: Sí, de hecho quiero decir que se aplica en todas partes. Tan triste, pero cierto, ¿verdad? Además, tampoco estoy abogando por hacer solo matemáticas puras en las que uno no tiene absolutamente ninguna confianza que tengan relevancia para resolver problemas en el mundo real. De todos modos, solo mi opinión. =)
Puede que esté leyendo mal esta publicación, pero parece haber una sugerencia de que no vale la pena seguir una carrera académica en matemáticas puras si uno es mediocre o de segunda categoría. No estoy seguro de cómo me siento acerca de esa sugerencia. Tampoco se menciona esa otra parte de ser profesor, a saber, enseñar. Esa omisión me inquieta: hay razones distintas a la investigación para considerar la academia.
@JesseMadnick Cuando era estudiante de posgrado, me disuadían activamente de enseñar. Cuando pasé más tiempo preparando notas de recitación y ejemplos interactivos para un curso de probabilidad en el que trabajaba como asistente de enseñanza, mi asesor me llevó a un lado y me reprendió por no dedicar más tiempo a la investigación. Era la misma historia que escuché de mis compañeros y de mis muchos contactos y amigos en la academia en otras universidades y en muchas disciplinas diferentes (no solo matemáticas o ciencias).
Cuando combina esto con artículos como "El académico desechable" de The Economist y observa las estadísticas sobre la tasa de crecimiento del personal adjunto que se espera que asuma la mayor parte de las tareas docentes, mientras se les paga mucho menos, dado que los paquetes de beneficios son menos completos. , y casi ninguna oportunidad para avanzar en la carrera, de hecho creo que es muy imprudente dedicarse a la academia por el bien de ingresar a la enseñanza de nivel universitario. Es casi como una lotería. Tal vez 1/1000 de los mejores graduados de doctorado tengan una carrera docente satisfactoria. El resto publica o perece y se va a la industria.
@EMS, ¿qué significa exactamente "separación inestable"?
Aquí quiero decir casualmente que nadie "está cerca" del límite. Si eres lo suficientemente bueno en matemáticas como para que lograr matemáticas profesionales muy difíciles no te impida tener un buen equilibrio entre el trabajo y la vida, entonces tu habilidad matemática es mucho, mucho más alta que incluso un doctorado en matemáticas promedio. graduarse de las mejores universidades. Por otro lado, si no tienes una destreza matemática tan extrema, pero aun así trabajas obstinadamente tanto como sea necesario para hacer matemáticas avanzadas para una carrera, entonces es casi seguro que no tienes un buen equilibrio entre el trabajo y la vida. Por supuesto que puede haber excepciones; Afirmo que son muy raros.
Me disculpo por la tardanza y espero que esto no se desvíe demasiado del tema, pero ¿cree que este consejo también se aplica a muchos aspirantes a físicos?
¿Estás diciendo que la mala memoria no te permitirá estudiar matemáticas?
El simple trabajo duro en matemáticas no funcionará.
Debo decirte que cambié por completo mi impresión sobre ti después de leer tu publicación aquí. Una excelente.
Respuesta un tanto peligrosa, ya que demasiadas personas abandonan las matemáticas debido a problemas psicológicos y fobia a las matemáticas. Es cierto que siempre hay genios como Abel o Galois, pero si otros campos también se vuelven elitistas como este, el mundo hubiera sido peor.

Algunos consejos no matemáticos muy básicos y lo siento si sueno como tu madre. Si sientes que tu memoria es mala y no estás encontrando suficiente tiempo para socializar, quizás no estés encontrando suficiente tiempo para comer bien. Comer mucha fruta fresca, verduras frescas, pescado fresco, aceite de oliva y cereales le dará a su cuerpo los componentes básicos para hacer lo mejor posible. Se sabe que el pescado azul en particular es bueno para el cerebro. http://www.newscientist.com/article/mg20827801.300-mental-muscle-six-ways-to-boost-your-brain.html

Encuentro que el tiempo que paso cocinando/lavando es placentero mentalmente relajante tiempo de inactividad durante el cual surgen algunas de mis mejores ideas. Tal vez podría combinar esto con un aspecto social e invitar a la gente a tomar el té si está cocinando algo rico. Evite el alcohol que suele acompañar a estas situaciones si tiene la intención de volver al trabajo después.

Me gusta mucho esta respuesta. La importancia de una dieta adecuada (y no solo una dieta de estimulantes) no se puede exagerar. La fruta es especialmente importante, por supuesto, porque el cerebro funciona con azúcares, y la fruta está llena de ellos. +1
¡Los dulces también están llenos de azúcar! (cae en coma hipoglucémico...)

Con respecto a lo de "y tener un estilo de vida saludable". Bueno, también tienes que aprender cuándo parar.

A veces te das cuenta de que no entiendes algo y no tienes tiempo en ese momento para entenderlo. Trate de "encajonar" eso tanto como sea posible. Averigua la forma general del tipo de cosa que no entiendes. ¿Cómo funciona? ¿En qué contexto se usa esta idea? ¿Qué tipo de "entradas" tiene? ¿Aparece algo como por arte de magia? ¿Qué? ¿Habías visto algo así antes? Si tiene en mente estas vagas ideas, es muy posible que lo descubra mientras duerme, en una conversación con alguien, tal vez semanas después, tal vez años. Depende de la cosa en particular.

A veces no entiendes algo porque esa cosa es una completa tontería. Los profesores a veces dicen tonterías: son seres humanos y cometen errores. Los libros cometen errores. Recuerdo pasar mucho tiempo tratando de completar una prueba para un problema de tarea y todo lo que intenté falló. Una hora antes de la fecha de entrega de la tarea, estaba hablando con otra persona en clase. Me mostró un ejemplo que inventó para demostrar el teorema y lo reinterpreté como un ejemplo que refutaba el teorema. Estas cosas pasan. Del mismo modo, muchos libros de texto tienen sutiles errores "perezosos". Si no está particularmente seguro de sí mismo, puede pasar horas sintiéndose frustrado con ese problema. Habla con la gente.

Tienes toda la razón sobre "aprender cuándo parar", pero cuando leo las notas, me coloco en la posición de que ninguna de esas cosas se sabe que es verdad, soy yo quien se supone que debo verificarlo, y luego ser capaz de reproducir la prueba a otros. Supongo que este es precisamente el tipo de trabajo que hacen los investigadores. Y en la investigación, un solo error desapercibido puede hacer que toda la teoría se desmorone como fichas de dominó. Si no puedo entender la mayoría de las pruebas ahora, cuando los resultados se conozcan y se me presenten, ¿cómo podré descubrir otros nuevos?
Ayuda a construir "módulos" con los que te sientas cómodo y confiado. En muchas clases introductorias de variedades, muchas de las pruebas se reducen a algún tipo de comprensión del teorema de la función implícita o inversa. Por lo tanto, tener esos teoremas "codificados" es clave para hacer que la transición a la teoría múltiple sea un paso más modesto. Por lo tanto, su motivación en cierto sentido no debería ser llegar al punto en que pueda explicar una prueba a otra persona, sino llegar al punto en que la prueba le parezca natural. En muchos (la mayoría) de los temas, solo hay unas pocas ideas clave. Obtener un sentido de
y hacer las extrapolaciones naturales es clave. En ese sentido, encuentro que la tarea es muy valiosa.
+1 para "Si no tienes mucha confianza en ti mismo, puedes pasar horas frustrado por un problema así. Habla con la gente".
@LeonLampret: Cada formulario de prueba era nuevo en algún momento y fue descubierto (probablemente varias veces, antes de que se diera cuenta) por alguien que no lo sabía y nunca se lo había presentado. El descubrimiento de nuevas pruebas tiene que ver con una habilidad que no es "comprender las pruebas que se me presentan". No es su trabajo reproducir la prueba a otros, ese es el trabajo de las publicaciones. Finalmente, la publicación de trabajos incompletos produce resultados que se desmoronan como fichas de dominó. Has estado expuesto a los caminos trillados. En la frontera, los errores son comunes.
+1: 'Intenta "encajonar" eso tanto como sea posible'.

Sobre estudiar matemáticas:

Tu tiempo en esta roca es finito, la cantidad de conocimiento matemático es infinita. Debe elegir sabiamente sobre lo que quiere pasar su tiempo aprendiendo. Decide si quieres ser un tipo "maestro de todos, maestro de ninguno" o "maestro de uno, maestro de ninguno".

Personalmente, prefiero saber mucho sobre todo que todo sobre una sola cosa, por lo que diría que no pierdas el tiempo aprendiendo cada detalle. Aprecie el material de alto nivel, avance al siguiente campo hasta que se quede sin campos. Luego, trabaje hasta llegar al material de nivel inferior según lo permita su tiempo.

Tu aprendizaje no se detiene una vez que completas tu(s) título(s). Aprende a controlar tu propio ritmo.

En el lado del estilo de vida saludable de las cosas:

La mente sólo puede ser tan aguda como el cuerpo. Tómese el tiempo para hacer ejercicio, comer bien y dormir lo suficiente. Descubrirá que piensa con más claridad, retiene la información con mayor eficacia y es más feliz.

El equilibrio es de vital importancia. Solo tienes una vida para vivir y hay mucho más que matemáticas. Tómese el tiempo para explorar otros intereses, descubrir otros nuevos y volverse más completo. Cuanto mayor sea tu conocimiento general, mejor serás en matemáticas.

Vive la vida. Socialice, enamórese, corra un maratón, pelee, vaya al ballet, pinte una obra maestra, vaya a pescar, explore el mundo; haz las cosas que te hacen más que un simple matemático, haz las cosas que te hacen una persona.

Métete en una pelea. Como Evariste Galois , ¿no? :-)

Puedo ver que esta pregunta tiene un par de meses, pero me gustaría agregar algunos comentarios:

1) La mayoría de los matemáticos investigadores tienen mejor memoria y son más rápidos de lo que usted describe. Hay excepciones notables, pero debes entender que será difícil competir. Siempre tendrás que trabajar más duro que la mayoría de tus compañeros. Si le toma tres veces más tiempo corregir los exámenes, entonces este tiempo se perderá en su investigación a pesar de que podría tener el mismo talento para la investigación real. Por otro lado, no confíes en tus compañeros cuando solo dicen que entienden las cosas y son rápidos. En muchos lugares, es genial afirmar que aprobaste el examen con poco tiempo de estudio. A la larga, podrías superar a algunas de las personas que saben cómo aprender solo para un examen.

2) De su grupo de pares, un porcentaje muy pequeño se convertirá en investigadores. No tiene sentido compararte con personas que aprueban eficientemente los exámenes si quieres convertirte en investigador. Busque buenos estudiantes ambiciosos y socialice con ellos. Si son más rápidos y tienen mejor memoria que usted, entonces pregúnteles qué tiene de malo el Lema 3.4 cuya demostración le parece extraña. No tiene sentido encontrar todos los errores estúpidos usted mismo. Pregunta a tus compañeros, pregunta a tus profesores, pregunta aquí. Estás perdiendo el tiempo si tardas tres horas en descubrir que el profesor escribió "c" en lugar de "e".

3) Si te concentras demasiado en los detalles, tienes que entrenar los resúmenes. ¿Puedes explicarle a un estudiante principiante muy talentoso lo que aprenderá en álgebra lineal y análisis? en 10 oraciones? ¿En un par de minutos? ¿En un par de horas? ¿Sin papel? Cuando necesito el resultado de una conferencia que escuché como estudiante hace 15 años, no necesitorecordar las condiciones del teorema. Necesito darme cuenta de que este teorema es probablemente aplicable a mi problema, en qué conferencia o libro lo vi y luego puedo buscarlo para verificar si hubo alguna condición técnica que olvidé. Para hacerlo, tengo que recordar la esencia del teorema y la prueba, no los detalles. Además, si no entiende algo durante el aprendizaje, acepte preliminarmente el resultado, continúe y vuelva al resultado más tarde, no se detenga indefinidamente en una cosa.

4) Por lo general, la esencia es algo que a los profesores les gusta escuchar durante un examen oral. Verificarán los detalles aquí o allá, pero no necesitan escuchar los detalles ásperos todo el tiempo. ¿Está seguro de que está hablando con el nivel de detalle esperado durante sus exámenes orales? ¿O simplemente está asumiendo que el profesor quiere escuchar todos los detalles y comenzar de inmediato en el nivel épsilon? ¿Alguna vez has tratado de sentarte en los exámenes de otros estudiantes?

5) Busque estudiantes más jóvenes y ayúdelos a prepararse para sus exámenes (o responda preguntas aquí). Ayudar a los demás es el mejor método para mantener frescos los conocimientos adquiridos. Esto no será tiempo perdido.

6) Absolutamente no debes sacrificar tu salud física y mental. El sueño, la alimentación, el ejercicio, la vida social y los pasatiempos son importantes y no deben descuidarse durante un período de tiempo prolongado. ¿No es posible simplemente tomar menos conferencias por semestre? ¿A quién le importará después si te lleva un año más terminarlo? Algo tiene que ceder, y me parece que lo más fácil para ti es repartir el trabajo en más tiempo. (Y sí, me doy cuenta de que incluso sin matrícula hay altos costos de oportunidad, pero parece que ahora necesita más tiempo).

Estar en buena forma física te aclara la cabeza. Además, solo necesitas una hora de ejercicio al día para mantenerte en buena forma. ¡Haz algunas tareas múltiples! Caminar y pensar se puede hacer al mismo tiempo, entrenas tu memoria Y tu resistencia.

Resolver problemas matemáticos mentalmente es un buen ejercicio mental.

Uno de mis profesores tenía un conocido en la escuela de posgrado que llevó esto a un extremo. Los dos estaban sentados juntos tomando café cuando mi profesor comenzó a escribir algo en una servilleta cuando su amigo interrumpió diciendo: "Los verdaderos matemáticos no usan papel". ¡Supongo que era un tipo inteligente y no estaba bromeando! Por supuesto que este es un caso marginal. Suerte con tus estudios OP
ver también [BDNF][1] [1]: ncbi.nlm.nih.gov/pubmed/21198979
Totalmente de acuerdo +1. Sin embargo, para el OP, recomiendo hacer ese juego de cartas Krypto con números de casas.

Si eliges estudiar matemáticas con tanto ahínco, como lo describes, significa que te debería gustar, y supongo que sí. Aún así, decir que consume todo tu tiempo me preocupa. Las matemáticas deberían ser un placer; al menos así es para mí. Las matemáticas no deberían ocupar todo tu tiempo, porque tu cerebro necesita descansar para que puedas procesar las cosas más fácilmente. Aquí hay algunos consejos:

  • al estudiar, elige el grado de detalle o profundidad que deseas recorrer. No resuelvas todas las pruebas de un libro cuando lo estudies. Elige lo que realmente te interesa y lo que necesitas para tu curso. Por lo general, un curso no cubre un libro completo, y para obtener buenas calificaciones no es necesario saber mucho más de lo que se ha enseñado en el curso. Algunas cosas se aclararán solo con el tiempo y la experiencia; los aprenderá para el examen, pero comprenderá el panorama completo en un período de tiempo más largo, tal vez años.

  • Aprende a relajarte todos los días. Tal vez soy un tipo perezoso, pero siempre encuentro tiempo para caminar, andar en bicicleta, para tocar el piano, para ver una película. Por ejemplo, me relajo al resolver problemas en este sitio, o por mi cuenta, problemas que no tienen nada que ver con lo que estoy estudiando en este momento. Tome al menos 8 horas de sueño por noche. Cuando te relajas, dejas al cerebro la oportunidad de poner todas las cosas en orden. Muchos matemáticos tenían ideas reveladoras mientras hacían cosas ordinarias. Un paseo por el parque te puede ayudar a entender un punto clave en una demostración, o puede surgir la solución a un problema al hacer algún deporte o alguna tarea de la casa (a mí me pasó más de una vez; la más divertida fue que resolvió un problema dado a una prueba de selección de equipo para la OMI en mi cabeza,

  • por lo general, puede concentrarse mejor si tiene un objetivo mejor que 'terminar el libro'. Por ejemplo, tome un artículo en el campo que está estudiando (tal vez un maestro pueda ayudarlo con eso) e intente comprender ese artículo en detalle. Estudie solo los teoremas y demostraciones que están relacionados con eso. Las matemáticas se han desarrollado enormemente en los últimos años. Intentar llevar el ritmo de todo es imposible. Centrarse en un ámbito más limitado suele ser más fácil, y en la investigación esto es realmente necesario.

  • No te preocupes nunca por memorizar todo. Olvidarás muchas cosas sin importar cuántas veces las aprendas, pero lo esencial que debes hacer es recordar dónde buscar las cosas que olvidaste. Por ejemplo: el teorema X con ejemplos y contraejemplos se presenta en el libro Y, el tema Z se puede encontrar en el libro T, y así sucesivamente. Intenta dividir tus pruebas en pasos que puedas recordar. No memorices los cálculos. Recuerde solo los puntos clave y confíe en que puede completar los espacios en blanco.

  • encontrar tiempo para pasarlo con amigos o colegas. Tener a alguien con quien compartir una idea, incluso matemática, puede ser de gran ayuda.

  • encuentre a alguien a quien pueda dar tutoría (a nivel de escuela secundaria o universidad, en un año inferior). Esto puede ser de gran ayuda económicamente y notarás lo bien que entiendes las cosas cuando tratas de explicárselas a alguien que no las conoce en absoluto. Esto ha sido de gran ayuda para mí.

Buena suerte.

Soy estudiante de grado en una universidad estadounidense y estoy pasando casi exactamente por lo que estás describiendo... la falta de sueño, falta de vida social, poca memoria para las demostraciones y una insistencia perfeccionista en que las matemáticas se presenten "a mi manera". Como digo, actualmente estoy pasando por esto, así que no puedo ofrecer ninguna respuesta. Sin embargo, tengo algunas sugerencias de la experiencia.


Recientemente me encontré en la desafortunada situación de tener que memorizar muchas demostraciones el día antes de un examen. De repente, ya no importaba si conocía solo unas pocas pruebas con gran detalle. Lo que necesitaba era conocer todas las pruebas, pero solo con suficiente detalle como para justificar un crédito parcial suficiente. Para hacer esto, hojeé las pruebas en mi libro de texto una por una, escribiendo pequeños resúmenes de cada una con mis propias palabras.

Mi punto es que esto (para mí) fue un método muy efectivo para captar las grandes ideas de las pruebas sin obsesionarse con los detalles. Al escribir mis propios resúmenes, también pude resumir pruebas completas en un par de oraciones, que luego sirvieron como mnemotecnia para la memorización.

Pero en cuanto a las preguntas que realmente hiciste...

¿Deberías saltarte la lectura de las pruebas? Lo ideal sería que los leyera y entendiera todos, pero si tiene poco tiempo (como parece), entonces tiene que ser eficiente. Ryan Budney tiene razón: tienes que aprender cuándo parar. Aprende lo que crees que es relevante para hacerlo bien en la clase. Luego, cuando termine el curso, puede tomarse el tiempo para comprender los detalles o las pruebas menos importantes o lo que quiera, si así lo desea.

¿Deberías esforzarte menos, obtener peores calificaciones, pero "tener una vida"? No creo que nadie pueda responder a eso excepto tú, me temo.

Sin embargo, diré que la eficiencia realmente importa, y que es posible que puedas encontrar formas de equilibrar lo académico con la vida social si las buscas. Ya sabes, de alguna manera todos somos bastante eficientes cuando llega el momento del examen, logrando acumular grandes cantidades de información en muy poco tiempo. No tenemos más remedio que ser eficientes. Entonces, aunque no digo que debas tratar todos los días como si fueran el día antes de un examen, sí creo que puedes encontrar formas de aumentar la eficiencia si las buscas.


Debo señalar que todo esto pretende ser un consejo práctico en lugar de un sabio consejo. Para un sabio consejo, también recomiendo el consejo profesional de Terrence Tao, así como hablar con sus profesores y asesores.

Finalmente, debo mencionar que tengo entendido que, aunque todavía no soy un matemático profesional, al final del día, la disciplina y el trabajo duro son tan importantes como el talento natural, si no más.

Entonces, si le preocupa poder producir matemáticas de nivel de investigación, entonces mi consejo sería que deje de preocuparse por eso. Si aún no ha probado suerte en la investigación, entonces no hay razón para preocuparse por ello prematuramente. Al menos, esto es lo que me dijo mi asesor cuando le presenté estas inquietudes el año pasado y, en realidad, ha sido uno de los mejores consejos que he recibido.

Entre los consejos profesionales de Terrence Tao, encontré que su descripción de las etapas pre-rigurosa, rigurosa y post-rigurosa de la educación matemática es particularmente relevante para mí. Suena relevante para ti también: terrytao.wordpress.com/career-advice/…
Y realmente he encontrado que es verdad, también. He estado haciendo un esfuerzo consciente recientemente para dejar la etapa "rigurosa" y pasar a la etapa "post-rigurosa". Y tengo que decir, de nuevo como menciona Ryan Budney, que estoy empezando a ver que surgen ciertos patrones en las pruebas. Creo que este tipo de reconocimiento de patrones le sucede a la mayoría de las personas eventualmente, ¡pero primero tienen que haber estado expuestos a suficiente material!
Muy interesante, me alegro de no estar solo en esto, y también buenos consejos. Durante los últimos días me he preguntado, si de alguna manera (no se rían, esto es serio) me transportaran permanentemente en el tiempo a los antiguos griegos, cuántas matemáticas podría enseñarles. Especialmente si no estaban 'comprando' mis pruebas. Podría haber estado decepcionado si me enterara...

Un amigo me envió un enlace a este post y me parece muy interesante. Una cosa que es particularmente interesante es tu convicción de que no tienes talento. ¿Cómo sabes si tienes talento o no? ¿Qué es el talento de todos modos? ¿Por qué estás tan seguro de que tus compañeros que obtienen una prueba más rápido o la recuerdan por más tiempo tienen más talento que tú?

Es muy difícil explicar qué es el talento matemático. La mayoría de los problemas matemáticos que vale la pena resolver y la mayoría de los teoremas que vale la pena probar toman años para resolver y probar, por lo que la velocidad o la memoria no serán tan útiles al resolver o probar estos teoremas. Creo que la mayoría de los humanos tienen suficiente memoria para almacenar toda la información necesaria para trabajar en problemas que valen la pena durante algunos años (tienen suficiente tiempo para hacerlo). La velocidad en un proyecto de tres o cuatro años casi nunca es útil (por supuesto, hay excepciones notables, pero en promedio diría que esto es irrelevante). Lo que en realidad es mucho más útil es la diligencia, que parece tener. Si puede permanecer con un problema después de unos meses de fracaso, entonces tiene las cualidades adecuadas, creo.

Una cosa que la gente menciona a menudo es la imaginación. Nunca puedes saber si lo tienes o no hasta que realmente empiezas a trabajar en los problemas. Puedes aprender un idioma rápidamente, pero después de hacerlo, es posible que nunca te conviertas en poeta. Las matemáticas de pregrado, de hecho, la mayoría de las matemáticas que se enseñan en las clases estándar, solo están desarrollando un lenguaje que algunas personas aprenden más rápido que otras y algunas recuerdan más que otras, pero lo que harán con eso no es algo que se enseñe en esas clases.

Yo diría que el talento matemático es, de hecho, esta imaginación. Decir que alguien tiene talento matemático es simplemente decir que la persona ve más conexiones y relaciones matemáticas entre objetos matemáticos que la mayoría de la gente. Una persona dotada para las matemáticas tiene una fuerte intuición sobre qué línea de pensamiento conducirá a nuevas y hermosas teorías y nuevos descubrimientos. La buena memoria y la velocidad para realizar cálculos matemáticos y operaciones lógicas a menudo se confunden con talento matemático. Por supuesto que es un talento, uno muy útil, pero yo no lo llamaría un talento matemático.

En cualquier caso, parece que estás muy lejos de un lugar en tu vida en el que realmente puedas determinar si tienes talento matemático o no. Puede acelerar su viaje para llegar allí inscribiéndose en proyectos de investigación en lugar de tomar cursos de matemáticas, que son los indicadores más engañosos del talento matemático. También puede tomar cursos de lectura e inscribirse en cursos de nivel más alto, como cursos de posgrado. También hay programas de investigación de verano en los que puede inscribirse.

En cuanto al estilo de vida saludable, no lo sé. De hecho, pienso en matemáticas cuando hago mi ejercicio. Eso es lo bueno de la profesión, pero tienes que enseñarte las cosas a ti mismo. Por ejemplo, puedes aprender a pensar en matemáticas cuando haces cosas diarias de rutina como lavar los platos. Pero me tomó un tiempo llegar a este tipo de estado mental, como estudiante universitario también tenía un estilo de vida poco saludable y no hacía mucho ejercicio pensando que era una pérdida de tiempo. ¡Pero no lo es! Puedes aprender a pensar en matemáticas mientras las haces y también es una forma de rejuvenecerte y lidiar con el estrés.

La parte de la vida social es difícil. Como investigador matemático, necesita una cantidad significativa de tiempo para usted mismo. Dudo que haya matemáticos investigadores por ahí con grandes círculos sociales. Pero con un poco de esfuerzo puedes tener suficiente gente a tu alrededor y estas personas, en la mayoría de los casos lo sé, suelen ser personas muy interesantes, inteligentes y motivadoras.

En cuanto a cómo aprender matemáticas, bueno, nunca se sabe. Creo que todos ponemos mucho énfasis en aprender las pruebas y los detalles, pero cuando subimos a un nivel superior nos damos cuenta de que realmente no entendimos la prueba. Entonces, en cierto sentido, en realidad es una actividad sin sentido aprender realmente la prueba de un teorema que has visto por primera vez. Probablemente hacer algún tipo de cosa circular en la que aprendes cosas y, a medida que avanzas, vuelves a cosas anteriores y las vuelves a aprender es mejor.

Pero creo que lo más importante es que descubras por qué quieres aprender estas pruebas tan bien. Hay muchas más pruebas para aprender y nunca las sabrás todas de memoria. De hecho, ¿qué es lo que quieres aprender de las matemáticas? Es mejor concentrarse en encontrar el área de matemáticas que le gusta y aprender esa materia con un buen profesor de manera circular, donde la materia se vuelve cada vez más sofisticada y las ideas importantes que se han dejado atrás se revisan de vez en cuando.

En resumen, no hay nada que temer y estoy seguro de que resolverá estas cosas por sí mismo.

Creo que el consejo de carrera de Terence Tao puede responder a su pregunta. Te recomiendo encarecidamente que lo leas.

Editar: Y también How to Think Like a Mathematician: A Companion to Undergraduate Mathematics de Kevin Houston .

hmm, no he encontrado nada que se relacione con mi problema real: cómo estudiar para entender matemáticas, tener actividades de ocio y terminar a tiempo. Porque solo ha sido un gran sufrimiento y trabajo duro hasta ahora...

Yo estaba (y todavía estoy en cierta medida) pasando por una fase muy similar hace algún tiempo. No soy particularmente bueno en todas las áreas temáticas principales de las matemáticas y soy preciso acerca de los detalles y hay momentos en que todo me abruma un poco. En esos momentos, me entrego a las matemáticas divertidas y solo trato de demostrar resultados divertidos que me atraigan, sin importar cuánto tiempo tome o cuán triviales parezcan. Esto me mantiene apegado a las matemáticas y al mismo tiempo me relaja. También tomo tiempo todos los días para actividades no matemáticas porque si no lo hago, demasiada actividad mental invariablemente me da dolor de cabeza.

Creo que hacer matemáticas es muy parecido a tocar música. Es un trabajo duro, pero de vez en cuando puedes tocar cualquier melodía que te relaje. Todo dicho y hecho, creo que es importante recordar exactamente por qué hacemos matemáticas: ¡porque es divertido!

Añadido después de ver el comentario:

Veo. Puedo relacionarme con mis días de graduado con eso. Los superé de alguna manera con mucha angustia, y lo que aprendí de eso fue esto: es importante que estudies la forma adecuada y esa forma adecuada es única para todos. En mi segundo semestre, recuerdo que realmente luché con Análisis complejo de Ahlfors (todavía estoy un poco preocupado por eso) y la razón fue que ese libro no estaba orientado a mi forma de estudiar, y no había tiempo para dar argumentos minuciosamente para todo. "se supone que está claro" en el libro. Más tarde, leí otro libro sobre Análisis Complejo (Brown & Churchill) y lo que sé del tema se debe en gran parte a eso. Esto se debe a que el segundo libro estaba más orientado a mi proceso de comprensión interna que Ahlfors.

Gracias por el consejo, pero creo que no entiendes la situación: no hay tiempo para matemáticas divertidas, no hay tiempo para libros o relajación, etc. Cada año, tengo 12 cursos, cada uno tiene notas de unas 100 páginas, que yo tienen que digerir de alguna manera, y ser capaces de reproducirse en los exámenes. Esto es horrible. Estaría feliz de 'sobrevivir' con buenas calificaciones y algo de tiempo libre/actividad física. Sé lo que quiero hacer, si tuviera tiempo. Son los cursos de matemáticas los que consumen todo mi tiempo. Si no permito que esto suceda y TOMO un poco de tiempo libre, las consecuencias serán que entenderé aún menos.

Sugiero transferirse a un programa donde pueda hacer que sus intereses coincidan tanto como sea posible con las cosas que debe estudiar.

A la mitad de mi primer año como estudiante me topé con el programa de matemáticas de honor en la Universidad de Alberta y me enganché bastante. El programa de honores enfatizó el rigor, la comprensión, la técnica, la visualización, la precisión, básicamente solo una base realmente sólida.

En mi segundo año como estudiante universitario, tuve un curso introductorio de ecuaciones diferenciales bastante desafortunado (obligatorio) en el que todo consistía en hacer girar la fórmula. Las pruebas y las ideas no se veían por ninguna parte. Ese curso fue bastante frustrante para mí: me pareció una oportunidad tan desperdiciada para comenzar a conectar los diversos hilos que habíamos estado desarrollando en análisis, álgebra lineal, álgebra, etc.

Descubrí que, como estudiante universitario, por lo general tenía mucho tiempo para hacer todo lo que se me pedía que hiciera. Hubo momentos en los que me metí en situaciones que estaban cerca de estar por encima de mi cabeza, pero todo salió bien. Si nunca te esfuerzas demasiado, nunca sabrás lo que es "demasiado duro". Así que es bueno descubrirlo. Creo que ayuda cuando las cosas que tienes que hacer son las cosas que quieres hacer. Si no lo son, puedes terminar perdiendo el tiempo aburriéndote. Es una buena lección para aprender cómo lograr cosas aburridas, ¡pero espero que no haya demasiadas en tu educación universitaria!

La transferencia a un programa diferente sería problemática. En mi país hay 2 universidades, y yo estoy en la más grande, cursando el programa de matemáticas puras. Espero que las cosas sean aún peores en la otra universidad. Parece que ir al extranjero presenta dificultades aún mayores. Especialmente si quiero eventualmente vivir aquí.
No esforzarme no es el problema, como puedes leer en mi publicación inicial. El problema es que me he estado esforzando durante varios años, para compensar la falta de talento y entender realmente las cosas, y he reducido la calidad de mi vida considerablemente. Eficiencia es lo que me falta. Sospecho firmemente que la mayoría de los compañeros de clase no leen las pruebas, o simplemente las miran, para tener una idea. ¿Es esta la manera de hacerlo?
¿Por qué no hablar con algunos profesores eslovenos y preguntarles si tuvieron problemas similares y cómo los solucionaron? Puedes ir a universidades fuera de Eslovenia y eventualmente conseguir un trabajo en Eslovenia. Conozco al menos a un profesor en tu departamento que hizo precisamente eso .
Como estudiante universitario, no me detuve en todas las pruebas que nos dieron en clase. Hice los requeridos en la tarea y presté mucha atención a los que encontré difíciles o de alguna manera informativos en clase; por ejemplo, recuerdo que mi fe en el axioma de elección se vio sacudida por la demostración del Urysohn Lemma en topología de conjunto de puntos. A pesar de que la prueba técnicamente tenía sentido, todavía me parecía mal .
¿Pasaste por todas las pruebas en todos los cursos? Tenemos 12 asignaturas por año, cada una tiene en promedio 80-100 páginas (40-50 hojas A4) en forma manuscrita. Si muchas de las notas son 'inestables' con muchos errores, tanto de tipo lapsus como más profundos, ¿cómo se supone que uno debe manejar esto, sin sacrificar una gran cantidad de tiempo en ello?
Como estudiante (y aún hasta cierto punto) rara vez tomo notas detalladas en clase. Mi estrategia fue condensar la presentación en un resumen de 2 o 3 líneas. Si hubiera una idea realmente novedosa, la escribiría en detalle, pero por lo general solo trataría de sacar la idea principal de la presentación. Pero teníamos libros de texto o copias impresas de notas de conferencias, por lo que había pocas razones para tomar notas detalladas. Conozco a muchas personas muy exitosas que escriben todo y lo revisan todo en detalle; todos somos diferentes en ese sentido.
"...los que encontré difíciles o de alguna manera informativos en clase..." ¿ Cómo puedo saber qué pruebas son informativas, sin leerlas y comprenderlas? A menudo no entiendo algo, luego me detengo en ello, hasta que se aclara. La mayoría de las veces, es solo una prueba, pero a veces descubro que fue crucial para toda la construcción/prueba. Era el punto central de todo, y si no lo hubiera leído, me habría perdido el punto por completo.
No estoy seguro de si esto es útil a corto plazo. Pero una vez que ve los aspectos "elementales" de las matemáticas en riguroso detalle y los ha absorbido, aquí estoy pensando en el cálculo de una sola variable a partir de una definición axiomática de los números reales (como en el libro Apostol), y conjuntos básicos, relaciones, teoría de grupos, conceptos básicos de anillos, álgebra lineal básica, representación de mapas lineales sobre una base. Una vez que dominas todo eso, comienza a ser más fácil notar los contornos de las pruebas y pensar más de arriba hacia abajo que de abajo hacia arriba, siguiendo cada paso linealmente.
Hmm, no estoy seguro de cómo son los exámenes teóricos (orales) en tu universidad, pero aquí, en muchas clases, dependiendo del profesor, se nos pide que demostremos esto y aquello, con bastante detalle. He absorbido las cosas de pregrado, pero aún así, no he encontrado ningún atajo/forma fácil de digerir las pruebas una por una.
¿De qué tipo de pruebas estás hablando? ¿Eres estudiante de posgrado? Antes había asumido que eras un estudiante universitario.
Umm, tenemos tres etapas: 1st=3yrs(diploma), 2nd=2yrs, 3rd=3yrs(doctorado). Estoy en la 2da etapa, primer año, pero llevo 5 años estudiando, no 4, porque cambié de programa de 'matemáticas aplicadas' a programa de 'matemáticas'. ¿Cómo puedo ir a un examen oral sin entender la mayoría de las demostraciones? Aquí, todos están tomando notas y (en su mayoría) no hay copias impresas de las notas de las conferencias . Por lo tanto, copio las notas de los demás, ya que realmente no puedo seguir lo que explica el profesor Y escribirlo correctamente en mi cuaderno.
Ah, y estaba hablando de demostraciones serias (al menos desde mi punto de vista): álgebra abstracta, topología algebraica, análisis de variedades, teoría de grafos, etc.
De acuerdo, creo que todos funcionan de manera diferente cuando se trata de este tipo de cosas. Me gusta mucho tomar exámenes orales, que me pongan en el lugar. Mi estrategia es no estudiar demasiado los detalles finos de las pruebas, sino repasar el esquema general, a veces una y otra vez hasta que yo mismo cree los detalles. Trato de evitar tanto como sea posible leer los argumentos quisquillosos de otras personas, ya que es muy fácil que la prueba de otra persona me parezca un desorden. Entonces, tal vez algo así podría ser una estrategia: trate de no seguir las pruebas de otras personas demasiado de cerca. Hágalos usted mismo.
"intenta no seguir demasiado de cerca las demostraciones de otras personas. Hazlas tú mismo". Me gusta absolutamente esta idea, PERO solo se aplica a pruebas simples. No hay forma de que pueda descifrar la prueba del teorema de Seifert van Kampen, o el teorema de Gauss-Bonnet. Tengo que leer la prueba del profesor, que a veces tiene 3 páginas, y hay numerosos detalles que no me quedan claros.
SvKT puede parecer poco intuitivo al principio, pero una vez que lo haya digerido realmente, descubrirá que es bastante simple. Es solo mucha "contabilidad" y seguimiento de detalles relativamente pequeños que generan una explosión de notación y todos los detalles técnicos reales. Tal vez mire otras pruebas: las de Hatcher y Peter May son grandes comienzos. Para GB, tal vez mire la prueba de Guillemin y Pollack (originalmente esta se debe a Hopf).
ejemplo: hicimos un ejercicio en el curso de topología algebraica: calcular π 1 del solenoide diádico. Tenemos un toro sólido T en R 3 y un homeomorfismo h : R 3 R 3 tal que h mapea el toro en su interior, dos veces envuelto. Ahora A 0 := T y A norte := h ( T ) . Por que es i norte A norte ( A 0 A 1 A 2 ) incluso no vacío?? Tal vez sea una pregunta tonta, pero parece que no puedo responderla de inmediato. Nadie hace este tipo de preguntas, pero las tengo por cientos...
"SvKT puede parecer poco intuitivo al principio, pero una vez que realmente lo has digerido..." Ese es mi punto, se necesita una gran cantidad de esfuerzo y tiempo para digerir la mayoría de las pruebas en mi facultad. Sospecho que muchos compañeros de clase no están revisando las pruebas y lo están haciendo bien, pero no puedo entender las cosas hasta que he pasado suficiente tiempo pensando en ello y digerido la mayoría de las pruebas. Ese es el problema. No puedo entender las cosas, hasta que he pasado una gran cantidad de tiempo digiriéndolas. Sólo entonces tengo una idea de las técnicas y métodos del campo. Por eso tengo un estilo de vida horrible...
Esa pregunta es demasiado vaga para ser una pregunta matemática. "dos veces envuelto" es demasiado vago, por ejemplo. Cualquier por qué no lo es h : T T ?
Sé que es vago, pero así lo hicimos en nuestra clase. No culpo al asistente, un enfoque más riguroso probablemente conduciría a serios problemas y tecnicismos. Sin embargo, es una intersección de una secuencia de subconjuntos cerrados A 1 A 2 A 3 R norte siempre no vacío?
De todos modos, realmente no estoy tratando de criticar nada, pero imagino que al investigar/descubrir verdades matemáticas, este tipo de detalles pueden ser cruciales.
@Leon: que la intersección de una colección anidada de subconjuntos cerrados no vacíos de R norte es un conjunto cerrado no vacío, este es uno de esos teoremas fundamentales en el análisis que se deriva directamente de la completitud. Este es uno de esos teoremas que fue ampliamente cubierto en el primer y segundo año de mi educación universitaria. Piensa en las sucesiones de Cauchy.
Desde R norte es completa, cada secuencia de cauchy tiene un límite. El límite está incluido en la intersección, ya que está cerrada. Supongo que elegimos un punto de cada A norte para obtener una secuencia a norte . ¿Cómo sabemos que es cauchy?
Oh, tenía una suposición equivocada. Los conjuntos deben estar anidados, no vacíos y compactos . Así que están contenidos en una caja grande. Si sustituye cerrado por compacto, podría tomar A norte ser el complemento de la bola abierta de radio norte centrado en el origen.
Sí, entonces funciona. Hmm, esto también funciona para el caso del solenoide diádico, ya que se cierra una imagen homeomorfa de un toro sólido. ¡Gracias! Probablemente por eso necesitamos h para asignar R 3 R 3 . De todos modos, este tipo de preguntas son cientos, cuando leo cosas, y nadie parece estar abordándolas. Especialmente en el. Por eso paso mucho tiempo tratando de corregir las pruebas, de 'hacer que funcionen' , por lo que apenas me queda tiempo...
¿Quizás estás complicando demasiado las cosas? ¿Está buscando pruebas de alta tecnología cuando las técnicas de su primer año como estudiante universitario encajen más directamente? Es difícil para mí decirlo desde esta distancia.
+1: 'Si nunca te esfuerzas demasiado, nunca sabrás lo que es "demasiado duro".'

Esta respuesta intentará abordar solo la primera parte de su pregunta. Cuando estaba haciendo el trabajo de pregrado, gané mucho peso ya que mi forma principal de hacer mi tarea era simplemente sentarme y comer papas fritas o algo mientras acercaba la nariz a la piedra de afilar. Esto, en combinación con la necesidad de estudiar constantemente, era realmente malo para mi cuerpo y también me causó algunos problemas de ansiedad en el futuro. Alrededor de mi cuarto año, comencé a hacer listas diarias de tareas pendientes que incluían pequeños fragmentos de cosas que no eran matemáticas: Encontré videos cortos de ejercicios (10-15 minutos) en YouTube que sabía que tenía tiempo para comprometerme. , e hice ese "100 Push up Challenge" que probablemente puedas encontrar a través de Google (no llegué allí, ¡pero me divertí mucho en el camino!).

Hasta el día de hoy, reservo tiempo para al menos 20 minutos de ejercicio cada noche (te sorprendería lo concentrado que estás después) y me siento significativamente mejor físicamente. Una vez que lo has estado haciendo durante aproximadamente un mes, se vuelve natural.

En cuanto al problema social, diferentes personas hacen cosas diferentes. Voy en bicicleta a la escuela, así que me uní a un grupo de ciclistas. También encontré una serie de otros clubes sociales en la ciudad (Chicago en ese momento) que hacían cosas que me gustaban. Me sorprendió la cantidad de personas que solo buscaban a otras personas con quienes hablar, ¡y no todo era ciencia!

Puede darse el caso de que esto no funcione para usted, pero quería compartir mi experiencia en caso de que alguien lo encontrara aunque sea un poco útil.

Gracias. En realidad, no subí de peso por ese hábito, sino por la falta de ejercicio y porque comer me da energía para no dormir y me quita el nerviosismo. Solía ​​ser muy deportista, entrenaba MMA y todo, y ahora todo se ha ido al polvo.
Además, tengo muchos intereses y actividades que estaba y habría estado haciendo ahora, si tuviera más tiempo. Y tenía una vida social perfectamente buena antes de la facultad. Estoy atrapado en el estrés horrible y el ritmo frenético de la vida, del cual no puedo escapar, sin sacrificar mi comprensión y mis calificaciones. Y para colmo, existe una clara posibilidad de que no pueda tener una carrera en matemáticas (académicas), ya que las oportunidades son muy limitadas y no tengo conexiones. Entonces todo parece ser un trabajo en vano, lo que disminuye aún más mi motivación.
Entiendo completamente tus últimos puntos aquí; Varias personas (¡incluyendo una famosa!) me han llamado un matemático mediocre y esto tendía a deprimirme. En los últimos años, he decidido que incluso si no hay lugar en la academia para mí, ciertamente hay lugar en otros lugares y ¡esto no significa que tenga que dejar de hacer matemáticas! Ese tipo de pensamiento me mantiene en marcha, creo. No sé qué más podrías hacer además de tratar de programar tus días o asumir una menor carga de trabajo. Sin embargo, no estoy seguro de si alguno de estos funciona para ti.

Podría considerar distinguir la comprensión de las matemáticas de los requisitos de sus clases. Si necesitas memorizar una prueba para un examen, para permanecer en la escuela, entonces hazlo... pero no lo confundas con comprensión o con resolver pruebas por ti mismo. Es posible que descubra que dividir una prueba en las partes que entiende y las partes que no entiende simplificará el proceso de hacer frente a la prueba.

¿Quién sabe? Tal vez al haber memorizado una demostración que no comprende, encontrará que puede pensar en ella mientras hace otra cosa, y tal vez comprenderla en un instante.

Sé que hay un tipo aquí que quiere disuadirte de que hagas matemáticas... Tiene toda la razón, la escuela de posgrado es muy difícil, los posdoctorados son una pesadilla... Lo que es perturbador es que intenta tenderte una trampa. para una vida cómoda... Tal vez esto no es lo que quieres. Una cosa que sé con certeza es que cualquier ser humano es "el capitán de su alma". Si quieres hacer algo, y si no te rindes, lo lograrás. Nada que valga la pena hacer viene fácilmente. Por eso existe tanta competencia por profesiones como la de matemático o cocinero. Porque a las personas que quieren hacerlo les encanta y para ellos no hay nada mejor que puedan estar haciendo. Sí, no te pagarán bien, sí, algún idiota va a montar un coche mejor que tú. Sin embargo, la última pregunta es, si quieres seguir tu pasión hasta el destino final donde te lleva, tal vez al abismo, o si no quieres intentarlo nunca y dejarte empujar por todo tipo de personas que te dirán lo que piensan que es la vida y cómo piensa que la vida debe ser vivida. Sé que de todos modos no escuchas a estas personas porque cuestionas incluso los objetos matemáticos, preguntas "¿por qué?" que es todo lo que necesita, junto con un enfoque práctico de la vida, para una carrera en matemáticas. Yo tampoco escuché. La misma gente me dijo que no llegaría a ninguna parte, que las matemáticas son demasiado competitivas, que ahí no hay sitio para mí. Sé que de todos modos no escuchas a estas personas porque cuestionas incluso los objetos matemáticos, preguntas "¿por qué?" que es todo lo que necesita, junto con un enfoque práctico de la vida, para una carrera en matemáticas. Yo tampoco escuché. La misma gente me dijo que no llegaría a ninguna parte, que las matemáticas son demasiado competitivas, que ahí no hay sitio para mí. Sé que de todos modos no escuchas a estas personas porque cuestionas incluso los objetos matemáticos, preguntas "¿por qué?" que es todo lo que necesita, junto con un enfoque práctico de la vida, para una carrera en matemáticas. Yo tampoco escuché. La misma gente me dijo que no llegaría a ninguna parte, que las matemáticas son demasiado competitivas, que ahí no hay sitio para mí.

A nivel práctico, no es normal que las matemáticas provoquen un aumento de peso; podría haber factores externos que debería investigar. Además, las matemáticas no deberían causar una pérdida de la vida social, ¿quizás tus amigos no eran los más adecuados para ti?

Con respecto a la academia, infórmese sobre los medios prácticos para permanecer y sobrevivir en la academia. Necesitará un plan de juego y una estrategia bien pensada siempre presente durante los próximos 3 a 5 años en cada momento de su carrera hasta que llegue a la titularidad. De lo contrario, podrías terminar sin trabajo. La "moneda" más importante en la academia son las publicaciones, su cantidad y su calidad. Debería incorporarlos en su plan muy pronto. El prestigio de la institución académica y de los profesores con los que has trabajado es importante en los inicios. Mire a los matemáticos exitosos (ajústese al nivel en el que quiere estar dentro de 10 años, pero siempre intente hacerlo un poco mejor que ellos), sus trabajos, la progresión de su carrera, etc. para tener una idea de lo que es necesario. Te deseo la mejor de las suertes,

Creo que el punto que no se planteó en los variados comentarios anteriores es: ¿cuál es la naturaleza de las matemáticas y cómo se debe hacer? Yo creo que para cualquier actividad humana hay que hablar de metodología , pero no hay tanta literatura al respecto. Aquí hay un artículo sobre esto, para que estés de acuerdo o en desacuerdo. También puede consultar los Prefacios de mi libro Topología y Groupoides , que puede consultar en el interior de Amazon.

Uno también tiene que explorar la naturaleza de los propios talentos en relación con el tema. ¡Un profesor aconsejó a sus alumnos que hicieran lo que les resultara más fácil!

A algunas personas les fascinan los problemas y la combinatoria. Me encuentro más interesado en las preguntas que he ideado yo mismo, en parte con la esperanza de que pueda satisfacer tres criterios:

  1. Nadie más ha pensado en ello.

  2. La respuesta no es técnicamente difícil.

  3. La respuesta es importante.

Todo esto puede parecer poco probable de encontrar, pero si no buscas no encuentras. Uno de mis alumnos, Derek Waller, dijo que le gustaba tener cien ideas. Si el 10% de tus ideas son buenas, ¡eso te da diez buenas ideas! ¡Prueba una maldita cosa tras otra!

El tema principal de mi investigación desde 1965 más o menos, a saber, los groupoides, surgió al escribir un libro sobre topología. Escribir matemáticas para hacer las cosas claras y elegantes, y escribir una y otra vez, puede hacerte ver que puede haber otra manera de hacer las cosas.

El compositor Ravel dijo que deberías copiar. Si tienes algo de originalidad, esto se nota. Si no, ¡no importa! De hecho, una idea original puede ocurrir solo después de copiarla varias veces, ya que la idea se ha metido en el cerebro.

Por lo tanto, hemos advertido a los estudiantes que se supone que una tesis tiene una "tesis". Por lo tanto, debe comenzar escribiendo los antecedentes de esa tesis. A medida que lo escriba, es posible que se encuentre con partes no del todo satisfactorias. Eso es un comienzo.

He tenido la suerte de encontrarme con este amplio y flexible programa de teoría de grupos de dimensiones superiores , que consistía en probar los usos de los groupoides de dimensiones superiores, en el espíritu de la teoría de grupos, y particularmente en relación con la teoría de la homotopía. Sospecho/sé que muchas "autoridades" lo consideraron una tontería, lo cual ha sido una desventaja. Por otro lado, mantuvo abajo la competencia.

Sospecho que mi talento es un sentido de la estructura matemática. Me gusta el comentario de Philip Hall: "Uno debe tratar de desarrollar el álgebra apropiada para la geometría, y no tratar de forzar la expresión algebraica de la geometría en un modo particular, simplemente porque está disponible".

He conocido a "investigadores" a quienes les resulta difícil escribir matemáticas o no lo hacen todo. Mi consejo es todo lo contrario: sigue escribiendo. Para un trabajo, escribes en la parte superior de una hoja de papel: Título. Autor. Introducción. Eso dice lo que pretendes hacer. Por supuesto, es posible que tenga dificultades con la Sección 1 sobre definiciones básicas, ¡pero siempre puede volver a ella más tarde!

El esquema de metodología del viejo ejército consistía en escribir: 1. Objetivos generales. 2. Situación actual. 3. Objetivo inmediato. 4. Método.

¡No empieces con "Método"!

No vi esta respuesta, así que me imagino que lo intentaré.

“Joven, en matemáticas no se entienden las cosas. Sólo te acostumbras a ellos." Juan von Neumann

Siempre me esforcé por entender las matemáticas mientras las aprendía, pero sé cuándo detenerme y pasar a aprender algo más. No creo que nadie pueda entender completamente las matemáticas o la física, todos los grandes matemáticos y físicos todavía tienen problemas de comprensión. Parece que el nivel de comprensión que buscas es demasiado metafísico. Algunos de los mejores matemáticos tenían una memoria increíble, esto sin duda les ayudó porque podían utilizarla para desarrollar análisis posteriores, etc.

"Además, mi problema es tener poca memoria. Olvido mucho".

¿Has probado a usar un programa de flashcards? Hay un programa donde puedes agregar preguntas. Si respondió una pregunta correctamente, el programa le volverá a hacer la pregunta en 2 días. Si responde correctamente en 2 días, le volverá a preguntar en 4 días. Después de eso, el intervalo crecerá a 8, 16 y así sucesivamente. Lo encontré útil. También puede establecer un intervalo personalizado. El programa acepta símbolos matemáticos.

Además, no agregue TODO lo que quiera recordar al programa. Recordarás muchas cosas de todos modos. El truco es averiguar cuáles necesitas agregar.


Con respecto a cómo puedes perder la tentación de hacer matemáticas todo el tiempo, me gustaría mencionar el concepto de utilidad marginal. La utilidad marginal de cualquier cosa (digamos, una onza de agua) es cuánto te beneficia adquirir una unidad adicional. Si estás deshidratado, la utilidad marginal de una onza de agua es muy alta. A medida que bebes más y más agua, su utilidad marginal disminuye. Lo mismo sucede con el conocimiento matemático. Incluso si ha estado consumiendo matemáticas nuevas durante años y está ansioso por obtener más, habrá un punto en el que su utilidad marginal disminuirá y podrá concentrarse en otras cosas fácilmente. Todavía no sabes cuál es ese punto, y tal vez no lo alcances ni en años, pero según mi experiencia, espero que lo alcances en algún momento.

En realidad, la sobrehidratación puede ser fatal, si bebes demasiado dañarás tu cuerpo quizás hasta el punto de la muerte. Para llevar el punto de utilidad marginal a casa, si haces demasiadas matemáticas, podrías terminar sin aprender nada en absoluto.

Algo podría ser útil cuando se entrena a los soldados de las fuerzas especiales, llamado Devil Training.

Parece que las buenas razones están relacionadas con trascender el límite extremo de capacidad para extender rápidamente la zona de confort y equiparse con prácticas enormemente difíciles en entornos muy complicados, NO fáciles.

Algunas partes son lo mismo que un buen entrenamiento matemático, uno podría absorber conocimientos amplios e importantes rápidamente y avanzar rápidamente, con prácticas intensivas que se realizarán en un "entorno" avanzado y complicado para entrenar habilidades elementales y avanzadas al mismo tiempo. Nunca te quedes en el nivel fácil, sigue adelante incluso si algunas habilidades/conocimientos no se absorben bien, ya que será mejor a medida que avancemos. Sólo hay una línea muy fina entre la Sabiduría y la Insensatez, las personas sabias absorben y avanzan rápido jugando con el conocimiento como un pez nadando y continúan profundizando el entendimiento; mientras que los tontos solo sufren el dolor pasivamente y repiten todo mecánicamente sin pensar.

Si bien algunas partes son aparentemente opuestas, en el entrenamiento militar es obedecer las órdenes, sin hacer preguntas, pero en matemáticas (ciencias naturales) es romper las órdenes, hacer preguntas sin cesar. Más un poco de pasión, curiosidad, disposición autoautomática para explorar y algo de suerte, puede generar un camino muy hermoso en el mundo matemático.

Como CPA que ahora regresa a la escuela para obtener una Maestría en Ciencias de la Computación (y, por lo tanto, algunos cursos de matemáticas en el camino), puedo relacionarme indirectamente con su problema; sin embargo, creo que mi perspectiva es relevante y muy bien puede ser de ayuda para usted.

Yo era como tú en mis estudios de licenciatura en contabilidad. Literalmente pasé cada momento despierto de mi día durante los últimos dos años de mis estudios, siete días a la semana, preparándome para mis clases. Al igual que usted, me mueve el "por qué"; no sólo el "cómo". También es cierto en el ámbito empresarial que la mayoría de los estudiantes no preguntan por qué una solución es verdadera; simplemente aceptan que es verdad y memorizan los pasos para llegar a la solución correspondiente.

A diferencia de ti, no me considero solo marginalmente talentoso o mediocre en el mejor de los casos; y, de hecho, lo animo a considerar verdaderamente el comentario de un posteador anterior que desafió su opinión autocrítica al decir que usted puede tener talento y, por lo tanto, no hacer este juicio prematuramente sobre usted. Proverbialmente: ¿Cómo puedo construir una casa si no creo que pueda también? No sería una pérdida de tiempo considerar cómo su creencia interiorizada sobre sí mismo puede estar afectando significativamente A) su rendimiento académico y B) su nivel de ansiedad personal. Por favor, tómese esto en serio.

Habiendo hecho ahora tales declaraciones, también me gustaría decir que desde un punto de vista práctico "llego" totalmente a donde estás ahora. Quiere la perspectiva del mañana, hoy, y por lo tanto asegurarse de no tomar una decisión tonta hoy que le costará mañana. Mi consejo es seguir la pasión más profunda de tu corazón hoy y confiar en "mañana" para resolver los demás detalles.

Por ejemplo, si hacer matemáticas es realmente la pasión suprema de tu vida, entonces comprométete con este camino independientemente de si te conviertes en profesor titular o te encuentras en algún área de matemáticas aplicadas... o incluso desarrollo de software. Todos deben hacer lo mejor que puedan y luchar por el mayor logro. Siempre serás más feliz esforzándote por dar lo mejor de ti y al mismo tiempo siguiendo la pasión más profunda del corazón.

Por supuesto, hay más en la vida que "X" (p. ej., matemáticas, en su caso), por lo que también debemos nutrir las otras partes de nuestro ser. Incluso nuestros deseos básicos nos enseñan esto en su insistencia cada veinticuatro horas para alimentar, descansar, limpiar y aliviar. ¿Y qué hay de nuestras necesidades más elevadas, emocionales y espirituales? ¿Es mera vanidad buscar humanidad (compañerismo)? Al igual que con las suposiciones inherentes a las matemáticas, nuestros deseos carnales y psicológicos también proporcionan suposiciones evidentes para ayudar en el proceso de construcción de nuestra existencia. Escuche la suya y actúe en consecuencia, pero comprenda que no todas las casas tienen la misma forma, tamaño y simetría; por lo tanto, puede requerir muy poco o mucho de tales variables "y" mientras resuelve el factor "x" de su vida.

Establezca sus prioridades. Consigue un calendario. Haz ejercicio durante media hora 3 veces a la semana. Esto es mejor que nada. No hagas nada que te vaya a desgastar por completo. Programa tiempo para pasar el rato con la gente. Programa otras prioridades como dormir. Luego, cualquier tiempo que quede, que probablemente sería bastante, haz matemáticas. Si todavía se siente de la misma manera, programe más ejercicio/sueño/tiempo para pasar el rato/lo que sea.

Mucha gente se olvida de las matemáticas. Quería estudiar cosas de álgebra. Tuve Real Analysis 1 y 2 (nivel de posgrado) un año y pasé tanto tiempo en ellos que no tuve tiempo para álgebra. No podía recordar las cosas de álgebra que eran realmente importantes. El análisis real ni siquiera es importante para mí. Debería haber priorizado mejor y dedicar menos tiempo al análisis real y más tiempo a cosas algebraicas.

Ese año también pasé mucho tiempo lejos de mi familia, lo cual fue terrible. Ahora básicamente trabajo de 8 a 5 y me voy a casa. Y, tal vez el sábado trabajo unas horas. A veces, trato de conseguir algo más de trabajo. Pero mi familia es más importante que estas matemáticas, así que no me importa si me toma más tiempo graduarme o si no me va tan bien.

No estoy tratando de ser grosero ni nada, pero a menudo no ayuda decir simplemente un imperativo como "programar tiempo para hacer ejercicio". Por lo general, las personas preguntan cómo programar ese tiempo.
@EMS No quiero ser grosero, pero mirando el contexto, puedes ver que lo que dije tenía sentido. Está diciendo que dedica todo su tiempo a las matemáticas. Le estaba diciendo que programe momentos en los que no esté haciendo matemáticas y que haga matemáticas el resto del tiempo. Luego, hace cosas que antes no hacía.
Correcto, pero eso no ayuda a explicar cómo dedicar tiempo a otras cosas. El cartel ni siquiera habría hecho la pregunta a menos que él o ella ya supieran que deberían dedicar tiempo a estas otras cosas. Pero eso es muy diferente a un curso de acción real que brinda detalles de implementación sobre cómo lograr que uno mismo se comprometa realmente con estas cosas. Un consejo más relevante sería estudiar el estado del arte de la autoayuda , cosas como la ecuación de la procrastinación o el compromiso previo.
@EMS Si tiene una respuesta tan buena, responda la pregunta usted mismo.
Lo hice incluso antes de comentar tu respuesta ( enlace ). Lo siento si mis comentarios te ofendieron. Simplemente pretendía ser una crítica constructiva y educada.

Eres tan bueno como tu eslabón más débil. Muchos estudiantes y personas que trabajan en matemáticas subestiman el poder de estar en buena forma. Tu capacidad de concentración aumenta y tu sueño suele ser mejor cuando haces ejercicio y comes sano. Obtuve muchas ideas después de correr unas pocas millas mientras recuperaba el aliento después de quedar atrapado en algo.

En cuanto a la vida social; la familia es importante, los amigos cercanos también. Nunca asistí a ningún evento "social" en la universidad, es simplemente destructivo, no hay nada que ganar emborrachándose y quedándose despierto toda la noche. Tal vez una cena con algunos estudiantes y tomar unas cervezas y simplemente relajarse.

Para la parte del aumento de peso: Lea algunos escritos de Arthur De Vany y revise su dieta. Las personas comen constantemente y esperan no aumentar de peso. De Vany sugiere que algunos días, después de un gran desayuno saludable, está bien no comer nada y acostarse con hambre. La parte de "ir a dormir con hambre" recibe grandes objeciones de la gente, pero nuestros cuerpos aún no han evolucionado de su estructura de cazadores-recolectores. Cuando los humanos cazaban, podían pasar días sin comer mucho. Nuestro metabolismo está orientado a darse un festín (comer en exceso) después de una buena captura y andar hambrientos hasta esa captura. Comer constantemente no es nuestro camino. Prueba esto y serás más feliz, tu cuerpo cuando tenga hambre se aliviará incluso, porque habrá una cosa menos en la que trabajar. Los días que no esté comiendo (después del desayuno),

Quizás uno de los mejores consejos que he escuchado con respecto a este tipo de dilema (no solo en matemáticas, sino para todas las prácticas), es que si no es divertido, entonces debes cambiarlo.

Sus intereses matemáticos suenan como si se hubieran convertido más en una obsesión que en una diversión. Las matemáticas se ajustan claramente a tu forma de pensar y te esfuerzas por sentirte satisfecho al comprenderlas, pero las has dejado ir demasiado lejos.

Dos puntos:

  1. No entenderás todo
  2. Está bien no entender todo

Las matemáticas son una herramienta, y rara vez un estilo de vida. Puede que le resulte extraño dedicar toda su vida a comprender y diseñar un juego perfecto de llaves. Si bien al hacerlo, es posible que se le ocurra un conjunto mejor, deja de lado todas las reparaciones, la construcción y otras cosas productivas que puede hacer con el conjunto que tiene.

Vaya a usar sus matemáticas y permita que la comprensión que obtiene de su uso impulse una mayor comprensión a lo largo de las prácticas que le parezcan interesantes. Si es divertido, hazlo, y si no es divertido, cambia las cosas. Hágalo todo matemático en el camino, simplemente no se quede atascado en las matemáticas.

La agonía de "¿Cómo estudiar matemáticas?" no es tan real como se plantea. Antes de ir a estudiar un tema, uno debe dedicar un diez por ciento de duración a hacer una estrategia de cómo completar el Capítulo. Nunca des nada por sentado en Matemáticas. No confíe en la demostración de un teorema dado en los libros con demasiados "claros". No confíe en la autoridad del profesor. Rara vez son ideales para seguir. Simplemente trate de descubrir su propia prueba, poniendo cada definición y resultados previos ante sus ojos escritos en una amplia hoja de papel. Nunca confíe en la memoria con respecto a la definición o declaración de un teorema complicado cuando lo use en algún lugar. En su mayoría engañan a un novato. Simplemente míralo sin vergüenza. De esta manera disfrutarás y adquirirás el hábito de probar cualquier cosa por sí sola. Solo en caso de emergencia lea algunas partes de la prueba. No te vuelvas loco por resolver problemas. Rara vez son un desafío para un estudiante que ha pasado por el procedimiento que he descrito para completar un Capítulo. Preserve las "pruebas descubiertas por usted" para futuras necesidades. Te sorprenderá observar que después de estudiar de esta manera te vuelves más audaz cada día. Incluso entonces, si tiene problemas, debe dedicar más tiempo a hacer que su requisito previo sea vibrante. Abandonar la idea de abandonar las matemáticas. Solo las personas afortunadas pasan su vida en Matemáticas. Los jóvenes no necesitan tanto ejercicio como los mayores. Tu prioridad son las matemáticas, nada más. Si te levantas de la mesa sin completar el trabajo, tendrás que gastar el doble de tiempo en recapitular la próxima vez que te sientes. De esta forma serás un gran perdedor a la larga. Así que deja la mesa de estudio solo después de completar la tarea en cuestión. Rara vez son un desafío para un estudiante que ha pasado por el procedimiento que he descrito para completar un Capítulo. Preserve las "pruebas descubiertas por usted" para futuras necesidades. Te sorprenderá observar que después de estudiar de esta manera te vuelves más audaz cada día. Incluso entonces, si tiene problemas, debe dedicar más tiempo a hacer que su requisito previo sea vibrante. Abandonar la idea de abandonar las matemáticas. Solo las personas afortunadas pasan su vida en Matemáticas. Los jóvenes no necesitan tanto ejercicio como los mayores. Tu prioridad son las matemáticas, nada más. Si te levantas de la mesa sin completar el trabajo, tendrás que gastar el doble de tiempo en recapitular la próxima vez que te sientes. De esta forma serás un gran perdedor a la larga. Así que deja la mesa de estudio solo después de completar la tarea en cuestión. Rara vez son un desafío para un estudiante que ha pasado por el procedimiento que he descrito para completar un Capítulo. Preserve las "pruebas descubiertas por usted" para futuras necesidades. Te sorprenderá observar que después de estudiar de esta manera te vuelves más audaz cada día. Incluso entonces, si tiene problemas, debe dedicar más tiempo a hacer que su requisito previo sea vibrante. Abandonar la idea de abandonar las matemáticas. Solo las personas afortunadas pasan su vida en Matemáticas. Los jóvenes no necesitan tanto ejercicio como los mayores. Tu prioridad son las matemáticas, nada más. Si te levantas de la mesa sin completar el trabajo, tendrás que gastar el doble de tiempo en recapitular la próxima vez que te sientes. De esta forma serás un gran perdedor a la larga. Así que deja la mesa de estudio solo después de completar la tarea en cuestión.